Cat study

¡Supera tus tareas y exámenes ahora con Quizwiz!

The nurse finds a client with schizophrenia lying under a bench in the hall. The client says, "God told me to lie here." What is the best response by the nurse? 1 "I didn't hear anyone talking; come with me to your room." 2 "What you heard was in your head; it was your imagination." 3 "Come to the dayroom and watch television; you'll feel better." 4 "God wouldn't tell you to lie there in the hall. God wants you to behave reasonably.

"I didn't hear anyone talking; come with me to your room.

The nurse suspects risk of dysfunctional uterine bleeding in an adolescent. Which statement made by the adolescent would support the nurse's suspicion? 1 "I am losing weight unintentionally." 2 "I take herbal medicine for menstrual pain." 3 "I have severe pain during my menstrual periods." 4 "I am excited to get pregnant and can't wait."

"I have severe pain during my menstrual periods."

A client is admitted to the birthing unit with uterine tenderness and minimal dark-red vaginal bleeding. She has a marginal abruptio placentae. The priority assessment includes fetal status, vital signs, skin color, and urine output. What additional assessment is essential? 1 Fundal height 2 Obstetric history 3 Time of the last meal 4 Family history of bleeding disorders

Fundal height

A nurse is performing an eye assessment in an older adult. The older adult is unable to see near objects. Which conditions may be suspected in the older adult? Select all that apply. 1 Cataract 2 Glaucoma 3 Hyperopia 4 Presbyopia 5 Macular degeneration

Hyperopia Presbyopia

A nurse is teaching the parents of a school-aged child with sickle cell anemia about ways to prevent sickling. What should the nurse explain as the primary cause of sickling? 1 Hypoxia 2 Hemodilution 3 Hypocalcemia 4 Hemoglobin

Hypoxia

A nurse is advising a parent to teach the child impulse control and cooperative behaviors to avoid the risks of altered growth and development. In which stage does the nurse teach the parent about this, according to Erikson's theory of psychosocial development? 1 Initiative versus Guilt 2 Intimacy versus Isolation 3 Autonomy versus Sense of Shame and Doubt 4 Generativity versus Self-Absorption and Stagnation

Initiative versus Guilt

A nurse is writing a teaching plan about osteoporosis. How would the nurse best explain what osteoporosis is? 1 It is avascular necrosis. 2 It is caused by pathologic fractures. 3 It is hyperplasia of osteoblasts. 4 It involves a decrease in bone substance

It involves a decrease in bone substance.

A monoamine oxidase inhibitor (MAOI) is prescribed, and the nurse is formulating a teaching plan. What will the nurse instruct the client to avoid while taking this drug? 1 Fermented foods 2 Prolonged sun exposure 3 Strenuous physical exercise 4 Over-the-counter antihistamine drugs

Fermented foods

A nurse is assessing an 18-month-old toddler with suspected developmental dysplasia of the left hip. In what position should the nurse place the toddler to elicit the Trendelenburg sign? 1 Standing on the affected leg 2 Supine with the back arched 3 Side-lying on the unaffected side 4 Sitting upright with the legs separated

Standing on the affected leg

In the immediate period after admission to the burn unit with severe burns, a 5-year-old child requests a drink of milk. What is the most appropriate nursing intervention? 1 Giving ice chips as desired 2 Permitting milk if it has been iced 3 Maintaining NPO status for 24 to 48 hours 4 Limiting oral fluid to 15 mL every 4 hours

Maintaining NPO status for 24 to 48 hours

The nurse is caring for a client who had a thyroidectomy. Which symptoms will the client exhibit if having a thyrotoxic crisis? 1 An increased pulse deficit 2 A decreased blood pressure 3 A decreased heart rate and respirations 4 An increased temperature and pulse rate

An increased temperature and pulse rate

Which finding indicates that a newborn has vernix caseosa? 1 Brown hair on the skin 2 Rosy to yellowish skin 3 Light-pink to reddish-brown skin 4 Cheese-like substance on the skin

Cheese-like substance on the skin

Which type of joint is present in between the client's tarsal bones? 1 Pivot joint 2 Hinge joint 3 Saddle joint 4 Gliding joint

Gliding joint

A 45-year-old client is scheduled to undergo a hysterectomy and expresses concern because she has heard from friends that she will experience severe symptoms of menopause after surgery. What is the nurse's most appropriate response? 1 "You're right, but there are medicines you can take that will ease the symptoms." 2 "Sometimes that happens in women of your age, but you don't need to worry about it right now." 3 "You should probably talk to your surgeon, because I am not allowed to discuss this with you." 4 "Women may experience symptoms of menopause if their ovaries are removed with their uterus."

"Women may experience symptoms of menopause if their ovaries are removed with their uterus."

Which clients should be considered for assessing the carotid pulse? Select all that apply 1 Client with cardiac arrest 2 Client indicated for Allen test 3 Client under physiologic shock 4 Client with impaired circulation to foot 5 Client with impaired circulation to hand

-Client with cardiac arrest -Client under physiologic shock

After assessing a 5-year-old child, the nurse suspects the child has mumps. Which symptoms present in the child may support the nurse's suspicion? Select all that apply. 1 Skin rash 2 Sore throat 3 Fever (100° F-104° F [37.8°-40° C]) 4 Difficulty swallowing 5 Swelling of the parotid gland

-Fever (100° F-104° F [37.8°-40° C]) -Swelling of the parotid gland

The nurse is caring for a preschooler diagnosed as suffering from frequent episodes of sleep terrors. Which statements describing the nature of sleep terrors does the nurse know to be true? Select all that apply. 1 It is followed by full waking. 2 It usually occurs 1 to 4 hours after falling asleep. 3 It takes place during rapid-eye movement (REM) sleep. 4 The child rapidly returns to sleep after an episode of sleep terrors. 5 The child is aware of and reassured by another's presence after an episode of sleep terrors.

-It usually occurs 1 to 4 hours after falling asleep. -The child rapidly returns to sleep after an episode of sleep terrors.

Which physiologic characteristics of newborns affect drug dosage considerations? Select all that apply. 1 A newborn's less regulated body temperature 2 Immature liver and kidneys 3 Thick and less permeable skin 4 Lungs with weaker mucous barriers 5 Bacteria-killing acid in the stomach

-A newborn's less regulated body temperature -Immature liver and kidneys -Lungs with weaker mucous barriers

A nurse is teaching a client about a sodium-restricted diet. Which foods should the nurse encourage the client to consume? Select all that apply 1 Fruits 2 Cold cuts 3 Condiments 4 Fresh vegetables 5 Processed foods

Fruits Fresh vegetables

Arrange the order of steps in bone healing. 1. Hematoma formation 2. Callus formation 3. Invasion of granulation tissue 4. Osteoblastic proliferation 5. Bone remodeling

13245

A registered nurse is supervising a student nurse while assessing a 70-year-old client who is receiving aminoglycoside therapy. Which statement about the client's condition requires correction? 1 "The client may have deterioration of the cochlea." 2 "The client may have thinning of the tympanic membrane." 3 "The client may have an inability to hear high-frequency sounds." 4 "The client may have an inability to differentiate between consonants.

2 "The client may have thinning of the tympanic membrane."

A client reports swelling of the scrotum with no pain. Which condition does the nurse anticipate in the client? 1 Chancroid 2 Hydrocele 3 Spermatocele 4 Incarcerated hernia

2 Hydrocele

A client has been taking the prescribed dose of clozapine. The nurse will assess the client for which life-threatening side effect of this drug? 1 Polycythemia 2 Agranulocytosis 3 Hypertensive crisis 4 Pseudoparkinsonism

Agranulocytosis

Which nursing assessment is most important for a large-for-gestational-age (LGA) infant of a diabetic mother (IDM)? 1- Temperature less than 98° F (36.6° C) 2-Heart rate of 110 beats/min 3- Blood glucose level less than 40 mg/dL (2.2 mmol/L) 4- Increasing bilirubin during the first 24 hours

Blood glucose level less than 40 mg/dL (2.2 mmol/L)

The mental health nurse is facilitating a therapy group. How can the nurse further develop trust among the members of the group? 1 By discussing the importance of their trusting one another 2 By revealing some personal data as an example of trusting behavior 3 By having group members reveal some personal information about themselves 4 By reminding group members about the need for confidentiality within the group

By reminding group members about the need for confidentiality within the group

Which solid food can be given to a 6-month-old infant? 1 Raw pieces of fruit 2 Crackers or Zwieback 3 Firmly cooked vegetable 4 Well-cooked table foods

Crackers or Zwieback

A nurse is caring for a client who is receiving serum albumin. What indicates that the albumin is effective? 1 Improved clotting of blood 2 Formation of red blood cells 3 Activation of white blood cells (WBCs 4 Effective cardiac output

Effective cardiac output

What role is the nurse expected to have in a community-based nursing practice if there is a sudden spread of malaria? 1 Educator 2 Collaborator 3 Epidemiologist 4 Client advocate

Epidemiologist

A client is admitted with paresis of the ciliary muscles of the left eye. What function should the nurse expect to be affected? 1 Closing the eyelids 2 Convergence of both eyes 3 Ability to discriminate colors 4 Focusing the lens on near objects

Focusing the lens on near objects

The nurse is caring for a client with deep partial-thickness burns who is receiving an opioid for pain management. What is the preferred mode of medication administration for this client? 1 Oral 2 Rectal 3 Intravenous 4 Intramuscular

Intravenous

A client with severe preeclampsia is receiving magnesium sulfate therapy. What is the priority nursing assessment as the nurse monitors this client's response to therapy? 1 Urine output 2 Respiratory rate 3 Deep tendon reflexes 4 Level of consciousness

Respiratory rate

Which should the school nurse include in a teaching session with elementary school students related to nutritional intake and activity? 1 Eating the same types of food each day 2 Playing video games every day after school 3 Choosing high-fat foods for an after school snack 4 Consuming foods that are nutrient-rich, such as fruits

Consuming foods that are nutrient-rich, such as fruits

A nurse is assessing a client with Parkinson disease. Which assessment finding indicates the presence of bradykinesia? 1 Intention tremor 2 Muscle flaccidity 3 Paralysis of the limbs 4 Lack of spontaneous movement

Lack of spontaneous movement

The left foot of a client with a history of intermittent claudication becomes increasingly cyanotic and numb. Gangrene of the left foot is diagnosed, and because of the high level of arterial insufficiency, an above-the-knee amputation (AKA) is scheduled. Which response by the client best demonstrates emotional readiness for the surgery? 1 Explains the goals of the procedure 2 Displays few signs of anticipatory grief 3 Participates in learning perioperative care 4 Verbalizes acceptance of permanent dependency needs

Participates in learning perioperative care

A postpartum woman treated with methylergonovine complained of dizziness, has low blood pressure, and has passed out. Which initial nursing action would help this client? 1 Document the findings 2 Encourage excess increased fluid intake 3 Advise to stop taking methylergonovine 4 Notify the primary healthcare provider

Notify the primary healthcare provider

Which clinical indicator is the nurse most likely to identify when completing a history and physical assessment of a client with complete heart block? 1 Syncope 2 Headache 3 Tachycardia 4 Hemiparesis

Syncope

The nurse provides nutritional counseling to the parents of a 6-month-old formula-fed infant who will begin eating solid foods. Which statement by a parent indicates understanding of the nurse's advice? 1 "I'll keep giving him formula instead of regular cow's milk." 2 "I'll buy plenty of pureed spinach so she gets enough iron." 3 "Using a natural sweetener like honey is better than using table sugar." 4 "Baby food is sterilized, so it's better to feed directly from the jar than from a bowl.

"I'll keep giving him formula instead of regular cow's milk.

The parents tell the nurse that their preschooler often awakes from sleep screaming in the middle of the night. The preschooler is not easily comforted and screams if the parents try to restrain the child. What does the nurse instruct the parents? 1 "Always read a story to the child before bedtime." 2 "Intervene only if necessary to protect the child from injury." 3 "Discuss counseling options with the primary health care provider." 4 "Try to wake the child and ask the child to describe the dream.

"Intervene only if necessary to protect the child from injury."

The parents of a school-aged child with leukemia ask the nurse why irradiation of the spine and skull is necessary. What is the most accurate response by the nurse? 1 "Radiation retards the growth of cells in the bone marrow of the cranium." 2 "This therapy decreases cerebral edema and prevents increased intracranial pressure." 3 "Leukemic cells may invade the nervous system, but the usual drugs are ineffective in the brain." 4 "Neoplastic drug therapy without radiation is effective in most cases, but this is a precautionary treatment."

"Leukemic cells may invade the nervous system, but the usual drugs are ineffective in the brain.

A nurse is obtaining a health history from a client who is known to be verbally abusive. The client tells the nurse, "You're ugly, and you're probably stupid, too. Why am I stuck with you as my nurse?" What is the best response by the nurse? 1 "It doesn't matter what you think, because I know I'm a capable nurse." 2 "Tell me more about why my caring for you today is so upsetting to you." 3 "If you like, I will arrange to switch assignments so you can have another nurse." 4 "You are talking inappropriately, so I'm going to leave and will come back when you stop being verbally abusive."

"You are talking inappropriately, so I'm going to leave and will come back when you stop being verbally abusive.

A depressed client cries when the family does not visit. What is the most therapeutic response by the nurse? 1 "It's difficult to realize that no one cares about you." 2 "Your family didn't visit, and now you're feeling rejected." 3 "It's terrible to have such negative thoughts about yourself." 4 "Your family members work—that's why they don't visit you."

"Your family didn't visit, and now you're feeling rejected.

The nurse assesses and reports a disproportionate head-to-chest ratio for a 6-month-old infant. Which information should the nurse prepare to share with the infant's parents as the most likely cause of this assessment data? 1 "Your infant most likely has an altered chest shape." 2 "Your infant is likely experiencing abnormal head growth." 3 "Your infant will most likely be diagnosed with brain cancer." 4 "Your infant most likely had inaccurate measurements during the last visit."

"Your infant is likely experiencing abnormal head growth.

A client who weighs 176 pounds (80 kg) is being immunosuppressed by daily maintenance doses of cyclosporine to prevent organ transplant rejection. The dose prescribed is 8 mg/kg each day. How many milligrams should the nurse plan to administer each day? Record your answer using a whole number. ___ mg

- 640

A registered nurse is teaching a nursing student about Piaget's theory of cognitive development. What information should the nurse provide about the sensorimotor period? Select all that apply. 1 "In this stage, the child learns that he or she is separate from his or her parents or favorite toy." 2 "In this stage, the child develops a schema or action pattern for dealing with his or her environment." 3 "In this stage, the child believes that non-living objects have realistic thoughts, wishes, and feelings." 4 "In this stage, the child learns about himself or herself and the environment through motor and reflex actions." 5 "In this stage, the child promotes his or her cognitive development and learns about the world through playing."

-"In this stage, the child learns that he or she is separate from his or her parents or favorite toy." -In this stage, the child develops a schema or action pattern for dealing with his or her environment." -"In this stage, the child learns about himself or herself and the environment through motor and reflex actions."

The nurse is performing physical assessments for children in a daycare center. Which children should require a head circumference in order to monitor growth patterns? Select all that apply. 1 -A 6-month-old infant who is breastfed 2- A 15-month-old toddler who has asthma 3- A 3-year-old child whose birthday was the day prior 4 -A 5-year-old who will attend kindergarten in the fall 5- An 8-year-old child who will begin playing soccer next week

-A 6-month-old infant who is breastfed -A 6-month-old infant who is breastfed -A 3-year-old child whose birthday was the day prior

A nursing instructor asks a nursing student to outline the factors that predispose adolescents to substance use. Which of these statements outlined by the student are correct? Select all that apply. 1 Adolescents believe that substance use makes them more mature. 2 Adolescents believe that substance use will improve appetite and sleep disturbances. 3 Adolescents believe that using mood-altering substances creates a sense of well-being 4 Adolescents believe that substance use will help them achieve increased levels of performance. 5 Adolescents think that using substances will help them cope with worsening performance in school

-Adolescents believe that substance use makes them more mature. -Adolescents believe that using mood-altering substances creates a sense of well-being. -Adolescents believe that substance use will help them achieve increased levels of performance.

Methylphenidate is prescribed for a 6-year-old boy with the diagnosis of attention deficithyperactivity disorder (ADHD). The nurse teaches the father about safe medication administration and concludes that the instructions have been understood when the father says that he should administer it at which time? 1 At bedtime 2 After breakfast 3 When the child gets hungry 4 When the child's behavior is out of control

-After breakfast Methylphenidate (Ritalin SR) may cause nausea, anorexia, and dry mouth, which interfere with appetite and adequate food intake; therefore it should be administered after the child has eaten breakfast. Methylphenidate is a cerebral stimulant that can interfere with sleep; it should not be administered within 6 hours of bedtime. It should be taken exactly as prescribed, not on an as-needed basi

The nurse is providing postoperative care to a client on the second day after the client had a coronary artery bypass surgery. When assessing the water-seal chamber of the chest drainage device, the nurse observes that the fluid no longer fluctuates. What should the nurse do? 1 Assess for obstructions in the chest tube 2 Increase the amount of continuous suction 3 Add sterile water to the water-seal chamber 4 Make preparations to remove the chest tube

-Assess for obstructions in the chest tube. Fluid in the water-seal chamber should rise and fall as the client breathes in and out (tidaling) until the lungs have expanded completely; a lack of tidaling on the second postoperative day indicates that the tube is obstructed. Increasing the amount of suction is contraindicated without a prescription because it can traumatize pleural tissue. The level of the fluid, as long as it covers the tube in the water-seal chamber, does not affect tidaling. The lungs will not be fully expanded on the second postoperative day; the chest tube will remain in place.

The nurse is preparing a teaching plan for clients receiving antitubercular medications. Which teaching plan needs correction? 1 Clients taking ethambutol should drink plenty of fluids. 2 Clients taking Isoniazid should take the drug with food. 3 Clients taking pyrazinamide should wear a hat while going out in the sun. 4 Clients taking rifampin should use other contraceptive methods even after stopping the medication.

-Clients taking Isoniazid should take the drug with food. Taking isoniazid with food should be corrected. The presence of food may slow or even prevent the absorption of isoniazid from the stomach. Therefore the client should take the medication on an empty stomach, either 1 hour before or 2 hours after eating. Ethambutol may increase uric acid formation. The client should drink plenty of water to reduce uric acid precipitation and kidney problems. Pyrazinamide is a photosensitive medication that may increase the risk of sunburn. Therefore the client should wear a hat and protect himself or herself from sun exposure. Rifampin may decrease the efficiency of oral contraceptives. Therefore, the nurse should instruct the client to use an additional method of contraception even after stopping the medication.

Which actions by the nurse help set the stage for a patient-centered interview during the first visit after admission to the healthcare facility? Select all that apply. 1 Close the door after entering the room. 2 Greet the client using his or her last name. 3 Open the curtains to allow plenty of light in the room. 4 Introduce oneself with a smile and explain the reason for the visit. 5 Obtain an authorization from the client after the interview

-Close the door after entering the room. -Greet the client using his or her last name -Introduce oneself with a smile and explain the reason for the visit.

An individual is bitten by a snake that belongs to the class of North American pit vipers. Which medication should be used as a nursing priority to treat the client? 1 Diazepam 2 Epinephrine 3 Acetaminophe 4 Crotalidae Polyvalent Immune Fab (CroFab)

-Crotalidae Polyvalent Immune Fab (CroFab). CroFab is used for treating a victim bitten by a North American pit viper as a part of hospital care. Diazepam is used for the treatment of black widow spider bite. Diazepam is used as a muscle relaxant. Epinephrine is used as a nursing priority for bees and wasps sting. Acetaminophen is used for the treatment of fever related to scorpion bite.

A client is admitted with head trauma after a fall. The client is being prepared for a supratentorial craniotomy with burr holes, and an intravenous infusion of mannitol is instituted. The nurse concludes that this medication primarily is given to do what? 1 Lower blood pressure 2 Prevent hypoglycemia 3 Increase cardiac output 4 Decrease fluid in the brain

-Decrease fluid in the brain Osmotic diuretics remove excessive cerebrospinal fluid (CSF), reducing intracranial pressure. Osmotic diuretics increase, not decrease, the blood pressure by increasing the fluid in the intravascular compartment. Osmotic diuretics do not directly influence blood glucose levels. Although there is an increase in cardiac output when the vascular bed expands as CSF is removed, it is not the primary purpose for administering the medication

A nurse is providing discharge teaching for a client who recently had surgery for an abdominal perineal resection of the colon and the creation of a colostomy. Which condition will the nurse share with the client for when to call the healthcare provider immediately? 1 Intestinal cramps during fluid inflow 2 Difficulty inserting the irrigation tube 3 Passage of flatus during expulsion of feces 4 An inability to complete the procedure in one hour

-Difficulty inserting the irrigation tube Difficulty with inserting the irrigation tube indicates stenosis of the stoma and should be reported to the healthcare provider. Intestinal cramps during fluid inflow is a common response. Flatus is always present in the bowel to some degree, and a colostomy irrigation will facilitate its expulsion. Inability to complete the procedure within one hour is not indicative of a medical problem; a colostomy irrigation usually can be completed in one hour, but some individuals may need more time

Which statements about acne in adolescents are true? Select all that apply. 1 Early acne occurs in the midface region. 2 Acne is more common in girls than boys. 3 Acne usually occurs in middle to late adolescence. 4 Intake of dairy products can contribute to acne severity. 5 Acneiform eruptions are predominant in young children

-Early acne occurs in the midface region. -Acne usually occurs in middle to late adolescence. -Intake of dairy products can contribute to acne severity

The nurse cares for a client with bipolar disorder who is receiving drug therapy. The laboratory report reveals that the client's serum sodium level is 132 mEq/L (132 mmol/L). Which drug might have led to this condition? 1 Lithium 2 Bupropion 3 Fluoxetine 4 Nortriptyline

-Fluoxetine A serum sodium level of 132mEq/L (132 mmol/L) indicates hyponatremia. Fluoxetine is a serotonin reuptake inhibitor that may lead to hyponatremia. Lithium is a mood stabilizer used to treat bipolar disorder; it does not lead to hyponatremia. Bupropion is an atypical antidepressant that does not cause hyponatremia. Nortriptyline is a tricyclic antidepressant used to treat bipolar disorder that does not lead to hyponatremia

A nursing instructor asks a nursing student to explain the teaching methods to be used for adolescents. Which statement by the student indicates a need for further teaching? Select all that apply. 1 Keep teaching sessions short 2 Use teaching as a collaborative activity 3 Use problem-solving to help adolescents make choices 4 Encourage them to learn together, using pictures and short stories 5 Help adolescents learn about feelings and the need for self-expression

-Keep teaching sessions short -Encourage them to learn together, using pictures and short stories

Which data should the nurse anticipate when conducting a developmental assessment for a 5-year-old client? Select all that apply. 1 Names coins correctly 2 Has a vocabulary of 1500 words 3 Participates in parallel play 4 Ties shoe laces independently 5 Has hand dominance established

-Names coins correctly -Ties shoe laces independently -Has hand dominance established

An expectant couple asks the nurse about the cause of low back pain in labor. The nurse replies that this pain occurs most often when the fetus is in what position? 1 Breech 2 Transverse 3 Occiput anterior 4 Occiput posterior

-Occiput posterior A persistent occiput posterior position causes intense back pain because of fetal compression of the maternal sacral nerves. The breech position is not associated with back pain. The transverse position does not usually cause back pain. Occiput anterior is the most common fetal position and does not cause back pain.

A pregnant client is prescribed heparin to prevent the risk of thromboembolism. Which adverse effects should the nurse anticipate with this medication? Select all that apply. 1 Osteoporosi 2 Suppress contractions in labor 3 Increased risk of serious bleeding 4 Stimulation of uterine contraction 5 Compression fractures of the spine

-Osteoporosis -Compression fractures of the spine

A client with a reddish-blue generalized skin alteration is hospitalized. Laboratory findings show an increase in the overall amount of hemoglobin. Which condition might the nurse suspect? 1 Albinism 2 Addison's disease 3 Polycythemia vera 4 Methemoglobinemia

-Polycythemia vera. The generalized reddish-blue skin alteration is occurring due to increased overall hemoglobin and may be associated with polycythemia vera.

A nurse is providing discharge medication teaching to a client who will be taking furosemide and digoxin after discharge from the hospital. What information is most important for the nurse to include in the teaching plan? 1 Maintenance of a low-potassium diet 2 Avoidance of foods high in cholesterol 3 Signs and symptoms of digoxin toxicity 4 Importance of an adequate intake and output

-Signs and symptoms of digoxin toxicity The risk of digoxin toxicity increases when the client is receiving digoxin and furosemide,

Which statement is true of the middle childhood stage according to Kohlberg's development of moral reasoning? Select all that apply. 1 The child is in the conventional level. 2 The child is in the post-conventional level. 3 The child develops a social contract orientation. 4 The child develops a good boy-nice girl orientation. 5 The child develops a punishment-obedience orientation.

-The child is in the conventional level. -The child develops a good boy-nice girl orientation.

Which points should be included when the mother of an epileptic child is being counseled? Select all that apply. 1 The child should wear a medical alert bracelet. 2 The child should be given valproic acid with milk to reduce gastric irritation. 3 The parent should keep a journal of signs and symptoms before, during, and after seizures. 4 The child should discontinue the drug immediately and the parent should notify the primary healthcare provider if a rash develops 5 The parent should understand that chewable forms of antiepileptic drugs are recommended for once-a-day administration.

-The child should wear a medical alert bracelet -The parent should keep a journal of signs and symptoms before, during, and after seizures. -The child should discontinue the drug immediately and the parent should notify the primary healthcare provider if a rash develops.

A client is receiving hydrochlorothiazide. What should the nurse monitor to best determine the effectiveness of the client's hydrochlorothiazide therapy? 1 Blood pressure 2 Decreasing edema 3 Serum sodium level 4 Urine specific gravity

1 Blood pressure

The nurse accompanies a 3-year-old child to the playroom. The toddler seems afraid to select a toy or activity. What age-appropriate play materials should the nurse offer? Select all that apply. 1 Plastic tea set 2 Mold and clay 3 Play telephone 4 Pencil and paper 5 Simple video game

1 Plastic tea set 2 Mold and clay 3 Play telephone

The parents of a toddler with newly diagnosed cystic fibrosis ask a nurse what causes the problems related to this disorder. What should the nurse consider about the primary pathologic process before responding? 1 Hyperactivity of the eccrine (sweat) glands 2 Hypoactivity of the autonomic nervous system 3 Mechanical obstruction of mucus-secreting glands 4 Atrophic changes in the mucosal lining of the intestines

3 Mechanical obstruction of mucus-secreting glands

A 16-year-old client has a blood pressure reading of 119/75. What is the approximate pulse pressure? Record your answer using a whole number. __________ mm Hg ✓ Ans- The difference between the systolic and diastolic pressure is called the pulse pressure. The given blood pressure is 119/75. The difference between 119 and 75 is 44

44

A client's cardiac monitor indicates multiple multifocal premature ventricular complexes (PVCs). The nurse expects that the treatment plan will include a prescription for which medication? 1 Amiodarone 2 Epinephrine 3 Methyldopa 4 Hydrochlorothiazide

Amiodarone

A client's relative asks the nurse what a cataract is. Which explanation should the nurse provide? 1 An opacity of the lens 2 A thin film over the cornea 3 A crystallization of the pupil 4 An increase in the density of the conjunctiva

An opacity of the lens

A nurse is reviewing the history, physical examination, and diagnostic test results of a client with colitis. What clinical findings are associated with this disorder? Select all that apply. 1 Anemia 2 Diarrhea 3 Hemoptysis 4 Abdominal cramps 5 Decreased white blood cells

Anemia Diarrhea Abdominal cramps

A client has an endotracheal tube and is receiving mechanical ventilation. Periodic suctioning is necessary, and the nurse follows a specific protocol when performing this procedure. Select in order of priority the nursing actions that should be taken when suctioning. 1. Insert the catheter without applying suction 2. Assess client's vital signs and lung sounds 3. Rotate the catheter while suction is applied 4. Administer oxygen via a ventilation bag

Ans- The nurse should first assess the client's vital signs and lung sounds to determine if suctioning is needed. Then 100% oxygen should be administered to compensate for the lack of oxygen intake during the suctioning process. Suctioning should not be applied during catheter insertion to limit trauma. Rotating the catheter during withdrawal ensures thorough removal of secretions

Windows in the recreation room of the adolescent psychiatric unit have been broken on numerous occasions. After a group discussion one of the adolescents confides that another adolescent client broke them. What should the nurse do when using an assertive intervention instead of aggressive confrontation? 1 Confront the adolescent openly in the group, using a controlled voice and maintaining direct eye contact. 2 Knock on the door of the adolescent's room and ask whether the adolescent would come out to talk about the situation. 3 Approach the adolescent when the client is alone and, after making direct eye contact, inquire about the involvement in these incidents. 4 Use a trusting approach toward the adolescent and imply that the staff doubts the adolescent's involvement but requests a denial for the record.

Approach the adolescent when the client is alone and, after making direct eye contact, inquire about the involvement in these incidents

A client who has been experiencing double vision, drooping of the eyelids, and fatigue visits the neurologic clinic. A diagnosis of myasthenia gravis is made, and the healthcare provider prescribes pyridostigmine. The nurse should teach the client that it is important to take this drug based on what schedule? 1 On an empty stomach 2 One hour before meals 3 According to muscle strength 4 At the exact time intervals prescribed

At the exact time intervals prescribed

During assessment, the nurse asks a client about developmental milestones such as the age at which thelarche and menarche occurred. The nurse determines that the client experienced pubertal delay. Which finding in the client's history supports the nurse's conclusion? 1 Weight increased by 8 to 12 kg. 2 Menarche occurred 2 years after thelarche. 3 Breast development occurred by 15 years of age. 4 Growth in height stopped 2 years after menarche.

Breast development occurred by 15 years of age.

A female adolescent complains of breast pain. Which antigonadotropic herb may alleviate breast pain by decreasing prolactin levels? 1 Catnip 2 Black haw 3 Bugleweed 4 Chaste tree fruit

Bugleweed Bugleweed is an herb used to decrease breast pain by decreasing prolactin levels and facilitating an antigonadotropic effect. Black haw and catnip are herbs that act as uterine antispasmodics. Chaste tree fruit also decreases breast pain by decreasing prolactin levels, but it is not antigonadotropic.

The nurse is caring for a client who is scheduled for an electrophysiology study (EPS) because of persistent ventricular tachycardia. Before the procedure the client is to receive a beta-blocker. What client's response during the procedure best indicates that the betablocker is working effectively? 1 Decreased anxiety 2 Reduced chest pain 3 Decreased heart rate 4 Increased blood pressure

Decreased heart rate

A female client reports excessive hair growth on the face and chest. The nurse suspects ovarian dysfunction. Which findings support this assessment? Select all that apply. 1 Deepened voice 2 Enlarged clitoris 3 Capillary fragility 4 Changes in fat distribution 5 Increased thyroid gland activity

Deepened voice Enlarged clitoris

In her eighth month of pregnancy, a 24-year-old client is brought to the hospital by the police, who were called when she barricaded herself in a ladies' restroom of a restaurant. During admission the client shouts, "Don't come near me! My stomach is filled with bombs, and I'll blow up this place if anyone comes near me." What does the nurse conclude that the client is exhibiting? 1 Ideas of reference 2 Loose associations 3 Delusional thinking 4 Tactile hallucinations

Delusional thinking Delusions are false fixed beliefs that have a minimal basis in reality. This is a somatic delusion. Ideas of reference are false beliefs that every statement or action of others relates to the individual. Loose associations are verbalizations that sound disjointed to the listener. Tactile hallucinations are false sensory perceptions of touch without external stimuli

A pregnant woman is diagnosed with intrauterine fetal death. What would be the drug of choice for labor induction? 1 Oxytocin 2 Clomiphene 3 Dinoprostone 4 Methylergonovine

Dinoprostone Dinoprostone is a uterine stimulant indicated for the uterine evacuation of fetal remains. Oxytocin is used to aid in uterine contractions during childbirth. Clomiphene is indicated for female infertility in some clients. Methylergonovine is an oxytocic ergot alkaloid used to reduce postpartum uterine hemorrhage.

A nurse is caring for a client exhibiting compulsive behaviors. The nurse concludes that the compulsive behavior usually incorporates the use of which defense mechanism? A nurse is caring for a client exhibiting compulsive behaviors. The nurse concludes that the compulsive behavior usually incorporates the use of which defense mechanism? 1 Projection 2 Regression 3 Displacement 4 Rationalization

Displacement Displacement is the unconscious redirection of an emotion from a threatening source to a nonthreatening source. Projection is the attribution of one's unacceptable feelings and thoughts to someone else. Regression is the return to an earlier, more comfortable level of behavior; it is a retreat from the present. Rationalization is the attempt to make unacceptable behavior or feelings acceptable by justifying the reasons for them

The nurse educates a client on decreasing the risk of developing antibiotic-resistant infections. Which statement made by the nurse will be most significant? 1 "Wash your hands frequently." 2 "Do not skip any dose of your antibiotics." 3 "Save the unfinished antibiotics for later use." 4 "Stop taking the antibiotics when you feel better."

Do not skip any dose of your antibiotics."

A nurse is administering hydroxyzine to a client. The nurse will monitor the client for which common side effects of this drug? 1 Ataxia and confusion 2 Drowsiness and dry mouth 3 Vertigo and impaired vision 4 Slurred speech and headache

Drowsiness and dry mouth

What are the pharmacokinetic reasons for drug sensitivity in infants? Select all that apply. 1 Small body size 2 Drug absorption 3 Renal drug excretion 4 Protein binding of drugs 5 Hepatic drug metabolism

Drug absorption Renal drug excretion Hepatic drug metabolism

A client at 36 hours' postpartum is being treated with subcutaneous enoxaparin for deep vein thrombosis of the left calf. Which client adaptation is of most concern to the nurse? 1 Dyspnea 2 Pulse rate of 62 beats/min 3 Blood pressure of 136/88 mm Hg 4 Homan sign in the left leg

Dyspnea

An older adult tells the nurse, "I regret so many of the choices I've made during my life." Which of Erikson's developmental conflicts has the client probably failed to accomplish? 1 Ego integrity versus despair 2 Identity versus role confusion 3 Generativity versus stagnation 4 Autonomy versus shame and doubt

Ego integrity versus despair

Which type of burn/injury may cause a client to have a cervical spine injury? 1 Electrical burns 2 Chemical burns 3 Inhalation injury 4 Cold thermal injury ✓

Electrical burns

A client is admitted to the hospital with the diagnosis of acute salmonellosis. Which priority medication will the nurse prepare to administer? 1 Opioids 2 Antacids 3 Electrolyte 4 Antidiarrheals

Electrolytes

A 16-year-old boy with a diagnosis of adolescent adjustment disorder and his family are beginning family therapy. What is the best initial nursing approach? 1 Setting long-term goals for the family 2 Letting the client express his feelings first 3 Having the parents explain their rationale for setting firm limits 4 Encouraging each family member to share how the problem is perceived

Encouraging each family member to share how the problem is perceived

A 6-year-old child with autism is nonverbal and makes limited eye contact. What should the nurse do initially to promote social interaction? 1 Encourage the child to sing songs with the nurse. 2 Engage in parallel play while sitting next to the child. 3 Provide opportunities for the child to play with other children. 4 Use therapeutic holding when the child does not respond to verbal interactions

Engage in parallel play while sitting next to the child.

The mother of an infant with Down syndrome asks the nurse what causes the disorder. Before responding, the nurse recalls that the genetic factor of Down syndrome results from what? 1 -An intrauterine infection 2 -An X-linked genetic disorder 3 -Extra chromosomal material 4- An autosomal recessive gene

Extra chromosomal material

During a survey, the community nurse meets a client who has not visited a gynecologist after the birth of her second child. The client says that her mother or sister never had annual gynecologic examinations. Which factor is influencing the client's health practice? 1 Spiritual belief 2 Family practices 3 Emotional factors 4 Cultural background

Family practices

A newborn experiences a hypothermic period while being bathed and having clothing changed. Once the hypothermic episode has been identified and treated, what is the next nursing action? 1 Feeding the infant 2 Requesting a complete blood count 3 Monitoring the infant for hyperthermia 4 Allowing the infant to rest undisturbed

Feeding the infant

A client who had surgery for a ruptured appendix develops peritonitis. Which clinical findings related to peritonitis should the nurse expect the client to exhibit? Select all that apply. 1 Fever 2 Hyperactivity 3 Extreme hunger 4 Urinary retention 5 Abdominal muscle rigidity

Fever Abdominal muscle rigidity

The nurse suspects the Jarisch-Herxheimer reaction in a client with syphilis who is on antibiotic therapy. Which symptoms in the client support the nurse's suspicion? Select all that apply. 1 Fever 2 Hypertension 3 Vasoconstriction 4 Generalized ache 5 Pain at the injection site

Fever Generalized ache Pain at the injection site

A nurse is assessing the growth and development of a 3-year-old child. What does the nurse expect the child to be able to do? 1 Hop on one foot. 2 Button a sweater. 3 Cup the hands to catch a ball. 4 Go upstairs while alternating the feet.

Go upstairs while alternating the feet.

What activity beyond the capabilities of a toddler does the nurse conducting an evaluation expect of a 4-year-old child? 1 Naming several colors 2 Using three- and four-word sentences 3 Asking for the definitions of new words 4 Having a vocabulary of about 1500 words

Having a vocabulary of about 1500 words

A client is admitted to a psychiatric hospital with the diagnosis of schizoid personality disorder. Which initial nursing intervention is a priority for this client? 1 Helping the client enter into group recreational activities 2 Convincing the client that the hospital staff is trying to help 3 Helping the client learn to trust the staff through selected experiences 4 Limiting the client's contact with others while in the hospital

Helping the client learn to trust the staff through selected experiences

A client with diabetic ketoacidosis who is receiving intravenous fluids and insulin complains of tingling and numbness of the fingers and toes and shortness of breath. The cardiac monitor shows the appearance of a U wave. What complication does the nurse suspect? 1 Hypokalemia 2 Hypoglycemia 3 Hypernatremia 4 Hypercalcemia

Hypokalemia

The nurse is assessing the client admitted with diabetic ketoacidosis. Which statement made by the client indicates a need for further education on sick day management? 1 "I need to stop taking my insulin when I am ill because I am not eating." 2 "I will check my urine for ketones when my blood sugar is over 250." 3 "I will try and take in Gatorade and water when I am sick." 4 "I will continue all my insulin including my glargine when I am sick

I need to stop taking my insulin when I am ill because I am not eating."

A nurse educates a group of mothers about the nutritional needs of preschoolers. What statement made by a mother indicates the need for further teaching? 1 "I should give my child a variety of foods." 2 "I should increase the protein in my child's diet." 3 "I should be concerned about my child's weight gain." 4 "I should limit my child's calorie intake to 1,200 per da

I should limit my child's calorie intake to 1,200 per day.

A nurse is teaching Hands Only Basic Life Support for adults in the community. What should the rescuer do first after determining that the person is not responding and the emergency medical system has been activated? 1 Identify the absence of pulse. 2 Give two rescue breaths with a CPR mask. 3 Perform the head tilt-chin lift maneuver. 4 Perform chest compression at a rate of 100/min

Identify the absence of pulse

Which intervention does the nurse implement to develop a caring relationship with the client's family? 1 Deciding healthcare options for the client 2 Identifying the client's family members and their roles 3 Declining to inform the client's family after performing a procedure 4 Refraining from discussing the client's health with the family

Identifying the client's family members and their roles

A nurse in the coronary care unit (CCU) identifies ventricular fibrillation on a client's cardiac monitor. What intervention is the priority? 1 Elective cardioversion 2 Immediate defibrillation 3 An intramuscular (IM) injection of digoxin 4 An intravenous (IV) line for emergency medications

Immediate defibrillation

While assessing a client's skin, the primary healthcare provider finds honey-colored crust surrounded by erythema. Which bacterial infection does the nurse anticipate in the client? 1 Impetigo 2 Folliculitis 3 Carbuncle 4 Erysipelas

Impetigo Impetigo is a bacterial infection that has a honey-colored crust surrounded by erythema. Folliculitis is a bacterial infection with a small pustule at the hair follicle opening with minimal erythema. Carbuncle is a bacterial infection in which many pustules appear in an erythematous area, most commonly at the nape of neck. Erysipelas is a bacterial infection in which a red, hot, sharply demarcated plaque is indurated and painful

A client is found to have a borderline personality disorder. What behavior does the nurse consider is most typical of these clients? 1-Inept 2- Eccentric 3- Impulsive 4- Dependent

Impulsive

A client is admitted to the cardiac care unit with a diagnosis of myocardial infarction. The client asks the nurse, "What is causing the pain I am having?" Which explanation of the cause of the pain is the most appropriate response by the nurse? 1 Compression of the heart muscle 2 Release of myocardial isoenzymes 3 Rapid vasodilation of the coronary arteries 4 Inadequate oxygenation of the myocardium

Inadequate oxygenation of the myocardium

The laboratory report of a client reveals increased levels of atrial natriuretic peptide. Which other finding does the nurse anticipate to find in the client? 1 Decreased urine output 2 Increased concentration of urine 3 Increased sodium excretion in urine 4 Decreased glomerular filtration rate

Increased sodium excretion in urine

A client reports fever, headache, extreme tiredness, dry cough, sore throat, runny nose, muscle aches, nausea, vomiting, and diarrhea. Which organism is responsible for this condition? 1-Influenza virus 2-Toxoplasma gondii 3- Human herpes virus-8 4 -Cryptosporidium muris

Influenza virus Fever, headache, extreme tiredness, dry cough, sore throat, runny nose, muscle aches, nausea, vomiting, and diarrhea are symptoms of influenza. Influenza is caused by the Influenza virus. Toxoplasma gondii causes fever, altered mental status, headache, and seizures. Human herpes virus-8 causes vascular lesions on the skin. Cryptosporidium muris causes watery diarrhea and weight loss.

The nurse is admitting a client to the unit after fetal death was confirmed by ultrasound. While initiating intravenous (IV) therapy the nurse notes blood continually oozing from the puncture site. What is the nurse's next action? 1- Restarting the line distal to the initial site 2 -Informing the primary healthcare provider of this finding 3 -Starting the prescribed infusion of oxytocin 4 -Placing an oxygen mask on the client and setting the flow rate at 8 L/mi

Informing the primary healthcare provider of this finding

Which nursing action is important when suctioning the secretions of a client with a tracheostomy? 1 Use a new sterile catheter with each insertion. 2 Initiate suction as the catheter is being withdrawn. 3 Insert the catheter until the cough reflex is stimulated. 4 Remove the inner cannula before inserting the suction catheter

Initiate suction as the catheter is being withdrawn.

A couple, married for 5 years, wants to start a family. When speaking with the nurse the husband says, "Well, I guess we're going to have to jump into bed three or four times a day, every day, until it works." What is the nurse's best response? 1 Telling them to continue intercourse as usual until conception occurs 2 Instructing them in the frequency and timing of intercourse to promote conception 3 Discouraging this because sperm production decreases with frequent sexual intercourse 4 Agreeing that the frequency of intercourse must increase but twice daily is sufficient to promote conception

Instructing them in the frequency and timing of intercourse to promote conception

A nurse plans to teach a client with type 1 diabetes about the use of an insulin pump. What information will the nurse include in client teaching? 1 Insulin pumps mimic the way a healthy pancreas works. 2 The insulin pump's needle should be changed every day. 3 Pumps are implanted in a subcutaneous pocket near the abdomen. 4 The insulin pump's advantage is that it only requires glucose monitoring once a day.

Insulin pumps mimic the way a healthy pancreas works.

A nurse is discussing areas of potential concern, such as anemia, with several parents of 6- month-old infants. What should the nurse tell them is the most common cause of anemia in 1-year-olds? 1 Thalassemia 2 Lead poisoning 3 Iron deficiency 4 Sickle shape of blood cells

Iron deficiency

After counseling an older widowed client, a nurse concludes that the grieving process has been successfully completed when the client does what? 1 Is able to plan to start new relationships 2 Talks about the deceased spouse at great length 3 Ignores the deceased spouse's less-than-perfect qualities 4 Decides to leave the deceased spouse's study as it was before the death

Is able to plan to start new relationships

The nurse is caring for a client with a body surface burn injury of 55%. Which information will the nurse consider when planning care for this client? 1 Is prone to poor healing because of a hypermetabolic state 2 Has a decreased risk of infection when in a hypermetabolic state 3 Needs a cool environment to decrease caloric need 4 Will need 20 calories/kg during the healing process

Is prone to poor healing because of a hypermetabolic state

After a difficult labor a client gives birth to a 9-lb (4 kg) boy who expires shortly afterward. That evening the client tearfully describes to the nurse her projected image of her son and what his future might have been. What is the nurse's most therapeutic response? 1 "I guess you wanted a son very much." 2 "It must be difficult to think of him now." 3 "I'm sure he would have been a wonderful child." 4 "If you dwell on this now, your grief will be harder to bear

It must be difficult to think of him now."

A client seeking advice regarding contraception asks a nurse to explain how an intrauterine device (IUD) prevents pregnancy. How should the nurse respond? 1 "It covers the entrance to the cervical os." 2 "The openings to the fallopian tubes are blocked." 3 "The sperm are kept from reaching the vagina." 4 "It produces a spermicidal intrauterine environment.

It produces a spermicidal intrauterine environment.

The nurse is caring for a client who is admitted with a crushing injury to the spinal cord above the level of phrenic nerve origin. What should the nurse consider about this type of injury when planning care? 1 Ventricular fibrillation 2 Vagus nerve dysfunction 3 Retention of sensation and paralysis of lower extremities 4 Lack of diaphragmatic contractions and respiratory paralysis

Lack of diaphragmatic contractions and respiratory paralysis

A nurse is teaching a class for staff members working in a group home about the cognitive development of children with cognitive impairments. What concept can these children probably learn the fastest? 1 Love versus hate 2 Life versus death 3 Large versus small 4 Right versus wrong

Large versus small

A client with multiple myeloma who is receiving the alkylating agent melphalan returns to the oncology clinic for a follow-up visit. For which side effect should the nurse monitor the client? 1 Hirsutism 2 Leukopenia 3 Constipation 4 Photosensitivity

Leukopenia

The nurse is assessing a client with severe nodule-forming rheumatoid arthritis for possible Felty syndrome. Which assessment findings are consistent with Felty syndrome? Select all that apply. 1 Itchy eyes 2 Dry mouth 3 Leukopenia 4 Splenomegaly 5 Photosensitivity

Leukopenia Splenomegaly

The nurse has administered lymphocyte immunoglobulin to a client. Which side effects are most likely to occur? Select all that apply 1 Leukopenia 2 Peptic ulcer 3 Tachycardia 4 Serum sickness 5 Urinary infection

Leukopenia Tachycardia Serum sickness

Which drug does the nurse recognize as an effective mood-stabilizing drug used in clients with bipolar disorder and in the acute treatment of mania and prevention of recurrent mania and depressive episodes? 1 Doxepin 2 Clozapine 3 Amitriptyline 4 Lithium carbonate

Lithium carbonate

Which should the nurse encourage for a school-age client diagnosed with a chronic illness to enhance a sense of accomplishment? 1 Wearing make-up 2 Making up missed work 3 Participating in sports activities 4 Participating in creative activities

Making up missed work Making up missed work is an activity the nurse can encourage to enhance a sense of accomplishment for a school-age client who is diagnosed with a chronic illness

After a cesarean birth a nurse performs fundal checks every 15 minutes. The nurse determines that the fundus is soft and boggy. What is the priority nursing action at this time? 1 Elevating the client's legs 2 Massaging the client's fundus 3 Increasing the client's oxytocin drip rate 4 Examining the client's perineum for bleeding

Massaging the client's fundus

During the progressive stage of shock, anaerobic metabolism occurs. Which complication should the nurse anticipate in this client? 1 Metabolic acidosis 2 Metabolic alkalosis 3 Respiratory acidosis 4 Respiratory alkalosis

Metabolic acidosis

A client diagnosed with Bell palsy has many questions about the course of the disorder. Which information should the nurse share with the client? 1 Cool compresses decrease facial involvement. 2 Pain occurs with transient ischemic attacks (TIAs). 3 Most clients recover from the effects in several weeks. 4 Body changes should be expected with residual effects

Most clients recover from the effects in several weeks.

A 1-month-old infant with a ventricular septal defect (VSD) is examined in the cardiology clinic. What sign related to this disorder does the nurse expect to find when assessing this infant? 1 Bradycardia at rest 2 Activity-related cyanosis 3 Bounding peripheral pulses 4 Murmur at the left sternal border

Murmur at the left sternal border A murmur at the left sternal border is the most characteristic finding in infants and children with a VSD. A left-to-right shunt is caused by the flow of blood from the higher pressure left ventricle to the lower pressure right ventricle. Children with VSDs generally have tachycardia and are often acyanotic. A bounding peripheral pulse is not a common finding in children with a VSD.

A client develops a nonhealing ulcer of a lower extremity and complains of leg cramps after walking short distances. The client asks the nurse what causes these leg pains. Which would be the best response by the nurse? 1 "Muscle weakness occurs in the legs because of a lack of exercise." 2 "Edema and cyanosis occur in the legs because they are dependent." 3 "Pain occurs in the legs while walking because there is a lack of oxygen to the muscles." 4 "Pressure occurs in the legs because of vasodilation and pooling of blood in the extremities."

Pain occurs in the legs while walking because there is a lack of oxygen to the muscles."

The parents of a 12-year-old boy with cystic fibrosis (CF) ask the nurse why he needs a glucose tolerance test. What information should the nurse consider before replying? 1 Pancreatic scarring predisposes the child to diabetes 2 The thickened mucus blocks the insulin-secreting glands. 3 The test reveals the degree to which the child adheres to the diet. 4 Adjustments of the dosage of pancreatic enzymes are based on the results of the test

Pancreatic scarring predisposes the child to diabetes. Pancreatic scarring affects the ability of the islets of Langerhans to produce insulin; about half of all children with CF have altered glucose tolerance. The endocrine glands, which produce insulin, are ductless and are not affected by the thickened mucus in the ducts. However, the general scarring throughout the pancreas does affect the insulin-producing glands. The glucose tolerance test is a measure of the body's ability to produce and metabolize carbohydrates, not a measure of the child's adherence to the diet. The dosage of pancreatic enzymes is based on food consumption, not the blood glucose level

A client with type 1 diabetes requests information about the differences between penlike insulin delivery devices and syringes. What information does the nurse provide about the penlike devices? 1 The penlike devices have a shorter injection time. 2 Penlike devices provide a more accurate dose delivery. 3 The penlike delivery system uses a smaller-gauge needle. 4 Penlike devices cost less by having reusable insulin cartridges.

Penlike devices provide a more accurate dose delivery.

A client at 40 weeks' gestation is admitted to the birthing unit in labor. During the initial examination the nurse uses Leopold maneuvers to palpate the abdomen. The purpose of this intervention is to assess the what? 1 Station of the fetus 2 Position of the fetus 3 Duration of the contractions 4 Frequency of the contractions

Position of the fetus

A client with chronic kidney disease is receiving medication to manage anemia. Which primary goal should the nurse include in the care plan from this information? 1 Prevention of uremic frost 2 Prevention of chronic fatigue 3 Prevention of tubular necrosis 4 Prevention of dependent edema

Prevention of chronic fatigue

A nurse anticipates that most clients with phobias will use which defense mechanisms? 1 Dissociation and denial 2 Introjection and sublimation 3 Projection and displacement 4 Substitution and reaction formation

Projection and displacement Clients with phobias cope with anxiety by placing it on specific persons, objects, or situations through displacement, projection, or both. The person with a phobia recognizes and admits the exaggerated fear as a real part of the self and does not deny it

The nurse is assessing a client who has syndrome of inappropriate antidiuretic hormone (SIADH). Which finding in the client is consistent with the diagnosis? 1 Preservation of salt 2 Retention of water 3 Decrease of vasopressin 4 Presence of pedal edema

Retention of water SIADH is manifested in the form of retention of free water. This is because of excessive secretion of vasopressin causing reabsorption of water in renal tubules. There is hyponatremia and dilution of serum sodium in SIADH. Decreased vasopressin is seen in diabetes insipidus. Generally pedal (dependent) edema is not seen in SIADH despite the water retention

A client arrives at the emergency room complaining of chest pain and dizziness. The client has a history of angina. The primary healthcare provider prescribes an electrocardiogram (ECG) and lab tests. A change in which component of the ECG tracing should the nurse recognize as the client actively having a myocardial infarction (MI)? 1 QRS complex 2 S-T segment 3 P wave 4 R wave

S-T segment In ECG tracing, the displacement of the S-T segment is caused by an active ischemic injury in the myocardium. The QRS complex, the P wave, and the R wave are not associated with an MI

A client's severe anxiety and panic are often considered "contagious." What action should be taken when a nurse's personal feelings of anxiety are increasing? 1 Refocusing the conversation to more pleasant topics 2 Saying to the client, "Calm down. You're making me anxious, too." 3 Saying, "Another staff member is coming in. I'll leave and come back later." 4 Remaining quiet so personal feelings of anxiety do not become apparent to the client

Saying, "Another staff member is coming in. I'll leave and come back later."

While assessing a client with acquired immunodeficiency syndrome (AIDS), the nurse suspects that the client has developed cryptococcosis. Which clinical manifestations support the nurse's suspicion of a cryptococcosis infection? Select all that apply. 1 Seizures 2 Dyspnea 3 Blurred vision 4 Neurologic deficits 5 Enlarged lymph nodes

Seizures Blurred vision Neurologic deficits Seizures, neurologic problems/deficits, and blurred vision are the manifestations of cryptococcosis. Cryptococcosis is a debilitating meningitis and can be a widely spread infection in clients who have AIDS. It is caused by Cryptococcus neoformans. Histoplasmosis is a respiratory infection caused by Histoplasma capsulatum, which progresses to widespread infection in a client with AIDS. The symptoms of histoplasmosis are dyspnea and enlarged lymph nodes.

The nurse is admitting a confused 80-year-old client to the mental health unit. Which is one factor associated with the aging process? 1 Slowing of responses 2 Changing of personality 3 Lowering of intelligence 4 Diminution of long-term memory

Slowing of responses

A nurse is caring for a client who is cachectic. What information about the function of adipose tissue in fat metabolism is necessary to better address the needs of this client? 1 Releases glucose for energy 2 Regulates cholesterol production 3 Uses lipoproteins for fat transport 4 Stores triglycerides for energy reserves

Stores triglycerides for energy reserves

A nurse is caring for a client who is addicted to opioids and who has undergone major surgery. The client is receiving methadone. What is the purpose of this medication? 1 Allows symptom-free termination of opioid addiction 2 Switches the user from illicit opioid use to use of a legal drug 3 Provides postoperative pain control without causing opioid dependence 4 Counteracts the depressive effects of long-term opioid use on thoracic muscles

Switches the user from illicit opioid use to use of a legal drug

A nurse in the emergency department is assessing a client who has been physically and sexually assaulted. What is the nurse's priority during assessment? 1 The family's feelings about the attack 2 The client's feelings of social isolation 3 The client's ability to cope with the situation 4 Disturbance in the client's thought processes

The client's ability to cope with the situation

The nurse provides discharge teaching for a client with a history of hypertension who had a femoropopliteal bypass graft. Which client statement indicates teaching is effective? 1 "I should massage my calves and feet every day." 2 "I should keep my foot elevated when I am in bed." 3 "I should sit in a hot bath for half an hour twice a day." 4 "I should observe the color and pulses of my legs every day."

"I should observe the color and pulses of my legs every day."

The registered nurse is teaching a nursing student about ways to minimize heat radiation. Which statements made by the nursing student indicate effective learning? Select all that apply. 1 "I will apply an ice pack to the client." 2 "I will cover the client with dark clothes." 3 "I will instruct the client to remove extra clothes." 4 "I will instruct the client to lie in the fetal position." 5 "I will advise the client to wear sparsely woven clothes

"I will apply an ice pack to the client." "I will cover the client with dark clothes." "I will instruct the client to lie in the fetal position.

A client is admitted to the psychiatric unit with the diagnosis of obsessive-compulsive disorder. The client washes her hands more than 20 times a day, and they are raw and bloody. What defense mechanism does the nurse conclude that the client is using to ease anxiety? 1 Undoing 2 Projection 3 Introjection 4 Displacement

Undoing Undoing is an act that partially negates a previous one; the client is using this defense mechanism to atone for unacceptable acts or wishes. The client is not attributing selfthoughts or impulses to another person or group, which is called projection. The client is not absorbing into the self a hated or loved object (introjection). Displacement is the transferring of feelings from one person, object, or experience onto another, less threatening person, object, or experience.

An infant is receiving 22 mL/hr intravenous fluid by way of an infusion pump. How much fluid will be infused between 8:00 AM and 8:00 PM? Record your answer as a whole number. ___ mL

- 264

Which stages would the nurse explain that a toddler goes through, according to Freud's theory? Select all that apply. 1 Oral 2 Anal 3 Phallic 4 Genital 5 Latency

1 Oral 2 Anal

Which clinical manifestation occurs in a client with adrenal insufficiency? 1 Vitiligo 2 Moon face 3 Hypertension 4 Truncal obesity

1 Vitiligo

A client undergoes dilation and curettage (D & C) after an early miscarriage (spontaneous abortion). The nurse finds her crying later in the day. What is the most appropriate statement by the nurse at this time? 1 "This must be a very difficult experience for you to deal with." 2 "You'll have other children to take the place of the one you lost." 3 "Of course you're sad now, but at least you know you can get pregnant." 4 "I know how you feel, but when a woman miscarries, it's usually for the best."

"This must be a very difficult experience for you to deal with."

A nurse educates parents about how to communicate with their 14-year-old. Which statement should the nurse make? 1 "You should ask your child closed-ended questions." 2 "You should avoid involving other individuals and resources." 3 "You should avoid discussing sensitive issues with your child." 4 "You should look for the meaning behind your child's words or action

"You should ask your child closed-ended questions."

What instructions should the nurse give to an adolescent to prevent sexually transmitted infections? Select all that apply. 1 "Remember to use condoms properly." 2 "Abstain from any kind of sexual activity." 3 "Make sure you are up-to-date with your vaccinations." 4 "Have sexual contact only if you and your partner are monogamous." 5 "Remember to have regular screenings for sexually transmitted disease."

-"Remember to use condoms properly." "-Make sure you are up-to-date with your vaccinations." -"Have sexual contact only if you and your partner are monogamous.

While auscultating the heart, a healthcare provider notices S3 heart sounds in four clients. Which client is at more risk for heart failure? 1 Child client 2 Pregnant client 3 Older adult client 4 Young adult client ✓

3 Older adult client

An older adult in an acute care setting is having urinary incontinence. Which interventions would help the client? Select all that apply. 1 Provide nutritional support 2Provide voiding opportunities 3 Avoid indwelling catheterization 4 Provide beverages and snacks frequently 5 Promote measures to prevent skin breakdown

2 Provide voiding opportunities 3 Avoid indwelling catheterization 5 Promote measures to prevent skin breakdown

What is the focus of the nurse's anticipatory guidance during the first trimester of pregnancy? 1 Birthing process 2 Signs of complications 3 Physical changes of pregnancy 4 Role transition into parenthood

3 Physical changes of pregnancy

A clinic nurse observes a 2-year-old client sitting alone, rocking and staring at a small, shiny top that she is spinning. Later the father relates his concerns, stating, "She pushes me away. She doesn't speak, and she only shows feelings when I take her top away. Is it something I've done?" What is the most therapeutic initial response by the nurse?

3- Telling the father that it is nothing he has done and sharing the nurse's observations of the child

The healthcare provider's prescription for intravenous fluid states that the client is to receive 1 L of fluid every 8 hours. If the equipment delivers 15 drops/mL, at what rate should the nurse regulate the flow? Record your answer using a whole number. ___ drops/minute

31

When assessing an 85-year-old client's vital signs, the nurse anticipates a number of changes in cardiac output that result from the aging process. Which finding is consistent with a pathologic condition rather than the aging process? 1 A pulse rate irregularity 2 Equal apical and radial pulse rates 3 A pulse rate of 60 beats per minute 4 An apical rate obtainable at the fifth intercostal space and midclavicular line

A pulse rate irregularity

A pregnant woman who is in the third trimester arrives in the emergency department with vaginal bleeding. She states that she snorted cocaine approximately 2 hours ago. Which complication does the nurse suspect as the cause of the bleeding? 1-Placenta previa 2- Tubal pregnancy 3 -Abruptio placentae 4- Spontaneous abortion

Abruptio placentae

The laboratory reports of a client who underwent a hypophysectomy show an intracranial pressure (ICP) of 20 mmHg. Which action made by the client is responsible for this condition? 1 Drinking lots of water 2 Eating high-fiber foods 3 Bending over at the waist 4 Bending knees when lowering body

Bending over at the waist Bending over at the waist should be avoided as this position increases intracranial pressure in clients who underwent hypophysectomy. Drinking lots of water and eating high-fiber foods reduce the risk of constipation, so this should not cause increased intracranial pressure. The client should bend the knees then lowering their body to reduce the risk of intracranial pressure.

A nurse assesses a client who had a gastric resection. During the first 24 hours after surgery, what symptom should the nurse expect to identify 1 Vomiting 2 Gastric distention 3 Intermittent periods of diarrhea 4 Bloody nasogastric drainage

Bloody nasogastric drainage

A client with acute kidney injury states, "Why am I twitching and my fingers and toes tingling?" Which process should the nurse consider when formulating a response to this client? 1 Acidosis 2 Calcium depletion 3 Potassium retention 4 Sodium chloride depletion

Calcium depletion In kidney failure, as the glomerular filtration rate decreases, phosphorus is retained. As hyperphosphatemia occurs, calcium is excreted. Calcium depletion hypocalcemia [1] [2] causes tetany, which causes twitching and tingling of the extremities, among other

A 2-month-old infant with the diagnosis of heart failure is discharged with a prescription for oral digoxin 0.05 mg every 12 hours. The bottle of digoxin is labeled "0.05 mg/mL." Which item should the nurse teach the mother to use when administering the medication? 1 Nipple 2 Calibrated syringe 3 Plastic measuring spoon 4 Bottle containing an ounce of water

Calibrated syringe

A school nurse is teaching high school girls regarding the importance of immunizations. Which newborn anomaly can occur if rubella is contracted during the first trimester of pregnancy? 1 Limb abnormalities 2 Hydrocephalus 3 Down syndrome 4 Cardiac anomalies

Cardiac anomalies

The nurse is caring for a client who reports excessive tearing. Which disorders does the nurse suspect could be responsible for the client's condition? Select all that apply. 1 Chalazion 2 Entropion 3 Hordeolum 4 Conjunctivitis 5 Keratoconjunctivitis sicca

Chalazion Entropion Conjunctivitis

A client with acquired immunodeficiency syndrome (AIDS) is receiving a treatment protocol that includes a protease inhibitor. When assessing the client's response to this drug, which common side effect should the nurse expect? 1 Diarrhea 2 Hypoglycemia 3 Paresthesias of the extremities 4 Seeing yellow halos around lights

Diarrhea

Which cognitive development is seen in early adolescence? 1 Having established abstract thoughts 2 Developing capacity for abstract thinking 3 Exploring the ability to attract opposite sex 4 Exploring a newfound ability for limited abstract thoughts

Exploring a newfound ability for limited abstract thoughts

After assessing a dark-skinned client, the nurse concludes that the client has cyanosis. Which assessment color variation helped the nurse reach this conclusion? 1 Grey color 2 Purple color 3 Dark red color 4 Purple-to-brownish color

Grey color

The family of a client with right ventricular heart failure expresses concern about the client's increasing abdominal girth. What physiologic change should the nurse consider when explaining the client's condition? 1 Loss of cellular constituents in blood 2 Rapid osmosis from tissue spaces to cells 3 Increased pressure within the circulatory system 4 Rapid diffusion of solutes and solvents into plasma

Increased pressure within the circulatory system

The community health care nurse uses emails to remind the community about vaccines and regular diabetic checkups. Which Quality and Safety Education for Nurses (QSEN) competency does the nurse address? 1 Safety 2 Informatics 3 Quality improvement 4 Evidence-based practice

Informatic

The nurse is explaining insulin needs to a client with gestational diabetes who is in her second trimester of pregnancy. Which information should the nurse give to this client? 1 Insulin needs will increase during the second trimester. 2 Insulin needs will decrease during the second trimester. 3 Insulin needs will not change during the second trimester. 4 Insulin will be switched to an oral antidiabetic medication during the second trimester

Insulin needs will increase during the second trimester.

A nurse plans care of 4-year-old hospitalized children on the basis of their developmental level. What is the major vulnerability of children this age? 1 Separation anxiety 2 Altered family roles 3 Intrusive procedures 4 Enforced dependency

Intrusive procedures

A nurse performs Leopold maneuvers on a newly admitted client in labor. Palpation reveals a soft, firm mass in the fundus; a firm, smooth mass on the mother's left side; several knobs and protrusions on the mother's right side; and a hard, round, movable mass in the pubic area with the brow on the right. On the basis of these findings, the nurse determines that the fetal position is what? 1 LOA 2 ROA 3 LMP 4 RMP

LOA

A pregnant woman reports severe headaches, chest pain, and fatigue. Upon diagnosis, the woman has hypertension. Which drug can be prescribed to reduce hypertension? 1 Lithium 2 Miglitol 3 Calcium gluconate 4 Magnesium sulfate

Magnesium sulfate

The nurse in the postanesthesia care unit is caring for a client who had a left-sided pneumonectomy. Which goal is priority? 1 Replace blood loss 2 Maintain ventilatory exchange 3 Maintain closed chest drainage 4 Replace supplemental oxygenation

Maintain ventilatory exchange

What is the priority nursing objective of the therapeutic psychiatric environment for a confused client? 1 Helping the client relate to other 2 Making the hospital atmosphere more homelike 3 Helping the client become accepted in a controlled setting 4-Maintaining the highest level of safe, independent function

Maintaining the highest level of safe, independent function

A healthcare provider prescribes one unit of whole blood for a client after gastrointestinal surgery. What is an important nursing action when administering blood? 1 Warm the blood to body temperature to prevent chilling the client. 2 Obtain baseline vital signs prior to beginning blood administration. 3 Draw a blood sample from the client before each unit is transfused. 4 Maintain patency of the intravenous catheter with dextrose solution

Obtain baseline vital signs prior to beginning blood administration

A client diagnosed with multiple myeloma has been given a poor prognosis. After discharge, the client plans to travel on an airplane and attend sporting events with friends and family. The nurse prepares a discharge teaching plan for this client. What should the plan include? 1 Eliminating travel plans to combat anemia-related fatigue 2 Reinforcing a positive mental attitude to improve prognosis 3 Preventing infection; the client is at risk for leukopenia 4 Restricting fluid intake; the client is at risk for congestive heart failure

Preventing infection; the client is at risk for leukopenia

A client develops thrombophlebitis in the right calf. Bed rest is prescribed, and an IV of heparin is initiated. What drug action will the nurse include when describing the purpose of this drug to the client? 1 Prevents extension of the clot 2 Reduces the size of the thrombus 3 Dissolves the blood clot in the vein 4 Facilitates absorption of red blood cells

Prevents extension of the clot

When providing preoperative teaching, what should the nurse focus primarily on? 1 Helping the client and family decide if surgery is necessary 2 Providing emotional support to the client and family 3 Giving minute-by-minute details of the surgery to the client and family 4 Providing general information to reduce client and family anxiety

Providing general information to reduce client and family anxiety

A nurse is admitting a 2-year-old toddler who ingested half of a bottle of aspirin tablets to the emergency department. What is the origin of the metabolic acidosis caused by aspirin toxicity? 1 Deep rapid breathing 2 Higher pH of gastric contents 3 Rapid absorption of salicylate 4 Increased renal excretion of bicarbonate

Rapid absorption of salicylate

A client with cancer of the prostate requests the urinal at frequent intervals but either does not void or voids in very small amounts. Which does the nurse conclude is most likely the causative factor? 1 Edema 2 Dysuria 3 Retention 4 Suppression

Retention

A client receiving morphine is being monitored by the nurse for signs and symptoms of overdose. Which clinical findings support a conclusion of overdose? Select all that apply. 1 Polyuria 2 Sedation 3 Bradycardia 4 Dilated pupils 5 Slow respirations

Sedation Bradycardia Slow respirations

A 15-year-old client tearfully states that her father has been sexually abusing her for the past 8 years. What statement should the nurse initially respond with? 1 "Which type of incidents preceded the abuse?" 2 "Sharing this information is a positive step in getting help." 3 "I have to report this to child protective services right now." 4 "What kinds of things does he do to you when he abuses you?"

Sharing this information is a positive step in getting help."

While completing an assessment, the nurse finds that a client has decreased thickness and excessive dryness of the epidermis. Which clinical finding is associated with this skin assessment? 1 Skin tears 2 Skin cancer 3 Skin fragility 4 Skin hyperplasia ✓

Skin fragility

After multiple upper respiratory infections, a school-aged child undergoes a tonsillectomy and adenoidectomy. Two weeks after surgery the nurse assesses the child's condition. On what should the nurse focus? Select all that apply. 1 Taste 2 Smell 3 Hearing 4 Breathing 5 Facial symmetry

Taste Smell Hearing Breathing

A client tells a nurse in the prenatal clinic that she has vaginal staining but no pain. Her history reveals amenorrhea for the last 2 months and pregnancy confirmation after her first missed period. What type of abortion is suspected? 1 Missed 2 Inevitable 3 Threatened 4 Incomplete

Threatened

Six hours after initiation of total parenteral nutrition, the client's serum glucose level increases to 240 mg/dL (13.3 mmol/L). What does the nurse conclude is the most likely cause of the increase? 1 The solution is too concentrated. 2 The infusion is flowing too rapidly. 3 The solution is exacerbating preexisting diabetes. 4 The infusion is too slow to meet total nutritional needs.

The infusion is flowing too rapidly.

After assessing a 4-day-old newborn, the nurse anticipates that the newborn has impaired vision. Which finding supports the nurse's conclusion? 1 The newborn blinks in response to light. 2 The newborn has visual acuity of 20/100. 3 The newborn does not produce tears while crying. 4 The newborn has no corneal reflex after a light touch.

The newborn has no corneal reflex after a light touch.

A nurse is caring for a client with hyperthyroidism. Which laboratory test will be most beneficial in monitoring the effectiveness of drug therapy? 1 Free thyroxine (FT4) 2 Thyroxine (T4), total 3 Free triiodothyronine (FT3) 4 Triiodothyronine (T3), total

Thyroxine (T4), total

A client has primary open-angle glaucoma. The nurse expects that the client will receive a prescription for which eyedrops? 1- Tetracaine 2 -Cyclopentolate 3- Timolol maleate 4- Atropine sulfate

Timolol maleate

Which statement is true about toddlers? 1 Toddlers grow about 4.2 cm each year. 2 Toddlers develop a sense of autonomy. 3 Toddlers are aware of the danger of water. 4 Toddlers gain approximately 2 to 4 pounds (0.9 to 1.8 kg) each year.

Toddlers develop a sense of autonomy.

Which intervention should the nurse provide while caring for an older adult client who is reported to have decreased estrogen production? 1 Use minimal tape on client's skin. 2 Cover the client with warm clothing. 3 Perform blood glucose test for the client 4 Monitor for bradycardia

Use minimal tape on client's skin

A client with bubonic plague has a body temperature of 103° F associated with chills, swollen glands, headache, and weakness. Which microorganism is most likely responsible for the client's condition? 1 Yersinia pestis 2 Bordetella pertusis 3 Mycobacterium tuberculosis 4 Corynebacterium diphtheria

Yersinia pestis

A woman who is frequently physically abused tells the nurse in the emergency department that it is her fault that her husband beats her. What is the most therapeutic response by the nurse? 1 "Maybe it was your husband's fault, too." 2 "I can't agree with that—no one should be beaten." 3"Tell me why you believe that you deserve to be beaten." 4 "You say that it was your fault—help me understand that.

You say that it was your fault—help me understand that."

A father asks a nurse for strategies to convince his 5-year-old to wear a helmet while bicycling. What should the nurse suggest to the father? 1 "You should forbid your child from riding a bicycle." 2 "You should wear your helmet while riding your bicycle." 3 "You should limit your child's bicycling to a defined area." 4 "You should tell your child about the risks associated with not wearing a helmet."

"You should wear your helmet while riding your bicycle.

A nurse is caring for a client who has had multiple myocardial infarctions and has now developed cardiogenic shock. Which clinical manifestation supports this diagnosis? 1 Cold, clammy skin 2 Slow, bounding pulse 3 Increased blood pressure 4 Hyperactive bowel sounds

Cold, clammy skin

What characteristic is most essential for the nurse caring for a client undergoing mental health care? 1 -Empathy 2-Sympathy 3 -Organization 4- Authoritarianism

Empathy

Which identity may fail to develop if the adolescent fails to feel a sense of belonging and acceptance? 1 Sexual identity 2 Group identity 3 Family identity 4 Health identity

Group identity

When visiting hours are over, a nurse approaches a client with paranoid schizophrenia, who shouts, "You're the one that made my lover leave me." What conclusion does the nurse make about the client?= 1 The patient is disoriented. 2 The patient is actively hallucinating. 3 The patient feels a sense of vulnerability. 4 The patient needs to have limits set after calming down

The patient feels a sense of vulnerability.

A nurse is counseling the spouse of a client who has a history of alcohol abuse. What does the nurse explain is the main reason for drinking alcohol in people with a long history of alcohol abuse

They are dependent on it.

A client is noted to have thickened toenails that overhang the toes. The registered nurse suspects a fungal infection and instructs the student nurse to examine the fungal infection to confirm the diagnosis. Which action of the student nurse needs correction? 1 Cutting the client's fingernails straight across 2 Using the client's fingernails for assessing capillary refill 3 Using the nail appearance alone for assessing fungal infection 4 Assessing skin next to the nail to determine whether the thick nail is irritating the skin ✓

Using the nail appearance alone for assessing fungal infection

A nurse is working with a client experiencing a major depressive episode. What is a longterm outcome for this client? 1 Talking openly about the depressed feelings 2 Identifying and using new defense mechanisms 3 Discussing the unconscious source of the anger 4 Verbalizing realistic perceptions of self and others

Verbalizing realistic perceptions of self and others A major part of depression involves an inability to accept the self as is, which leads to making demands on others to meet unrealistic needs. Talking about the client's depressed feelings is a short-term goal; looking at what is causing those feelings is a long-term goal. Developing new defense mechanisms is not the priority, because they tend to help the client avoid reality. Discussing the unconscious source of the anger is not important or crucial to the client's recovery.

New parents are asked to sign the consent form for their son to be circumcised. They ask for the nurse's opinion of the procedure. How should the nurse respond? 1 "Let's talk about it, because there are advantages and disadvantages. " 2 "It's a safe procedure, and it's best for male infants to be circumcised." 3 Although it may be a somewhat painful experience for the baby, I would allow it if I were you." 4 "You should talk to the primary health care provider about this if you have any questions."

"Let's talk about it, because there are advantages and disadvantages."

Which parental statement indicates the need for further education regarding the psychosocial development that occurs during infancy? 1 "My older kids are so excited that our 10-month-old can play hide-and-seek with them." 2 "Peek-a-boo is an appropriate activity to initiate with my baby around 9 months of age." 3 "I just bought my 6-month-old some new rattles to play with because they are easy to grasp." 4 "It is important that my baby develops trust so we always respond when he cries for us at night."

"My older kids are so excited that our 10-month-old can play hide-and-seek with them.

A client with mild preeclampsia is instructed to rest at home. She asks the nurse, "What do you mean by rest?" What is the most appropriate response? 1 "Tell me what you consider rest." 2 "Take three or four naps a day. " 3 "Stay off your feet as much as possible." 4 "Would you like to know what I think it means?"

"Tell me what you consider rest."

A client with chronic obstructive pulmonary disease is admitted to the hospital with a tentative diagnosis of pleuritis. When caring for this client, what should the nurse do? 1 Administer opioids frequently 2 Assess for signs of pneumonia 3 Give medication to suppress coughing 4 Limit fluid intake to prevent pulmonary edema

Assess for signs of pneumonia

When the fetal head begins to crown during an emergency precipitous birth, how should the nurse respond? 1 Pressing firmly on the fundus 2 Applying gentle perineal pressure 3 Encouraging the client to push forcefully 4 Telling the client to take prolonged deep breaths

Applying gentle perineal pressure

Which client is contraindicated for the use of uterine stimulants in early pregnancy? 1 Client with hypertension 2 Client with bleeding disorder 3 Client with cardiovascular problem 4 Client with thromboembolic disorder

Client with bleeding disorder

A 60-year-old woman is admitted for a vaginal hysterectomy and anterior and posterior repair of the vaginal wall. While taking the nursing history the nurse expects the client to state that one of the reasons she is having surgery is because she has been experiencing what? 1 Hematuria 2 Dysmenorrhea Pain on urination 4 Stress incontinence

Stress incontinence

A client with varicose veins is scheduled for surgery. Which clinical finding does the nurse expect to identify when assessing the lower extremities of this client? 1 Pallor 2 Ankle edema 3 Yellowed toenails 4 Diminished pedal pulses

Ankle edema

Which clinical findings are observed in a client suffering from an imbalance of adrenocorticotropic hormone? Select all that apply. 1 Anorexia 2 Hyponatremia 3 Slowed cognition 4 Postural hypotension 5 Decreased muscle strength

Anorexia Hyponatremia Postural hypotension

A client's serum lithium level is 0.2 mEq/L (0.2 mmol/L). What is the nurse's interpretation of this finding? 1 Within toxic range 2 Borderline toxic range 3 Below therapeutic range 4 Borderline therapeutic range

Below therapeutic range

What would the nurse describe as a similarity between the growth and development of toddlers and preschoolers? 1 Both gain 5 to 7 pounds per year. 2 Both need an equal amount of calories. 3 Both grow about 2.5 inches in height per year. 4 Both need at least 12 hours of sleep every night.

Both gain 5 to 7 pounds per year.

The nurse is reviewing the laboratory reports of a client who has sustained a significant reaction to the tuberculin skin test but has negative findings on bacteriologic studies. The reports further reveal the absence of x-ray findings compatible with tuberculosis (TB) and clinical evidence of TB. Which class of TB does the nurse suspect? 1 Class 0 2 Class 1 3 Class 2 4 Class 3

Class 2

A woman reports irregular menses, back pain, and weight gain. Upon diagnosis, the woman has low levels of estrogen and progesterone. Which drug may be prescribed if the woman wants to conceive? 1 Raloxifene 2 Methergine 3 Clomiphene 4 Dinoprostone

Clomiphene

A nurse is assessing a toddler with the diagnosis of lead poisoning. What is the most harmful adverse effect that the nurse anticipates? 1 Inadequate nutrition 2 Delayed development 3 Anemia and constipation 4 Renal and skeletal damage

Delayed development

A nurse is volunteering on the community crisis hotline. What is the final objective of the counseling process? 1-Reducing anxiety 2-Exploring feelings 3 -Developing constructive coping skills 4- Accomplishing the debriefing process

Developing constructive coping skills Past coping behaviors have been inadequate in resolving the current crisis; new coping skills are needed to manage anxiety-producing conflicts. Reduction of anxiety is an early objective. Exploration of feelings is an immediate objective. Accomplishment of the debriefing process is an early objective.

Which clinical manifestations in a client indicate hyperfunctional thyroid gland? Select all that apply. 1 -Anemia 2-Diarrhea 3 -Weight loss 4 -Decreased appetite 5- Distant heart sounds

Diarrhea Weight loss

A 28-year-old woman comes into the clinic and tells the nurse that she fears that she is infertile, because she has been trying to become pregnant unsuccessfully for 2 years. While collecting the health history the nurse learns that the client experiences irregular and infrequent menstrual periods. The client is overweight and has severe acne and alopecia. The primary healthcare provider diagnoses the condition polycystic ovarian syndrome (PCOS). Which of the following interventions is the most important? 1 Consoling the client over her inability to have children 2 Discussing weight loss, exercise, and a balanced low-fat diet 3 Providing information to the client on how to prepare for surgery 4 Informing the client that there are no long-term complications of PCOS

Discussing weight loss, exercise, and a balanced low-fat diet

Which suggestion should the nurse offer to parents who are concerned about caring for their toddler? 1 Refrain from giving more than five cups of milk a day. 2 Allow the toddler to choose a time to take medicine. 3 Let the toddler watch television if the parent is busy. 4 Encourage the toddler to drink from two-handled cups

Encourage the toddler to drink from two-handled cups.

The mother of an 11-month-old infant reports that the baby has allergies. After an assessment, the primary healthcare provider also suspects anemia. Which questions would the primary healthcare provider most likely ask the mother? Select all that apply. 1 Do you use 2% cow's milk? 2 Do you breastfeed? 3 Do you use whole cow's milk? 4 Do you use alternate milk products? 5 Do you provide 18 to 21 ounces of breast milk per day?

Do you use 2% cow's milk? Do you use whole cow's milk? Do you use alternate milk products?

What nursing action will most help a client obtain maximum benefits after postural drainage? 1 Administer oxygen as needed. 2 Encourage coughing deeply. 3 Place the client in a sitting position. 4 Encourage the client to rest for a half hour

Encourage coughing deeply.

A client has returned from surgery with a nephrostomy tube. Which is the most essential nursing intervention for this client? 1 Ensure free drainage of urine. 2 Milk the tube every 2 hours. 3 Keep an accurate record of intake and output. 4 Instill 12 mL of normal saline every 8 hours.

Ensure free drainage of urine.

A plan of care is created for a term small-for-gestational-age (SGA) neonate who has been admitted to the neonatal intensive care unit (NICU). The goal is for the newborn to reach 5 lb (2300 g) by a specified date. On the specified date the infant weighs 4 lb 2 oz (1871 g). What should the next step be in care planning for this infant? 1 Increase the daily number of calories 2 Change the goal to a more realistic number 3 Evaluate the problem before altering the plan 4 Postpone the evaluation date for another month

Evaluate the problem before altering the plan

What is the difference between evidence-based practice and quality improvement? 1 Evidence-based practice is a part of a regular clinical practice. 2 Evidence-based practice aims to improve client care and outcomes. 3 Evidence-based practice focuses on the implementation of evidence already known into practice. 4 Eviden

Evidence-based practice focuses on the implementation of evidence already known into practice.

A client is diagnosed with hyperthyroidism and is treated with 131I. Before discharge the nurse teaches the client to observe for signs and symptoms of therapy-induced hypothyroidism. Which signs and symptoms should be included in the teaching? Select all that apply. 1 Fatigue 2 Dry skin 3 Insomnia 4 Intolerance to heat 5 Progressive weight gain

Fatigue Dry skin Progressive weight gain

A nurse assesses a newly admitted client with a diagnosis of pulmonary tuberculosis (TB). Which clinical findings support this diagnosis? Select all that apply. 1-Fatigue 2- Polyphagia 3 -Hemoptysis 4 -Night sweats 5 -Black tongue

Fatigue Hemoptysis Night sweats

During a group discussion it is learned that a group member hid suicidal urges and committed suicide several days ago. What should the nurse leading the group be prepared to manage?

Fear by some members that their own suicidal urges may go unnoticed and that they may

The nurse is assessing a newborn of 33 weeks' gestation. Which sign alerts the nurse to notify the health care provider? 1 Flaring nares 2 Acrocyanosis 3 Heartbeat of 140 beats/min 4 Respirations of 40 breaths/min

Flaring nares

The nurse is assessing an older adult client with suspected hearing loss. Which observations made by the nurse in the client indicates a decrease in hearing acuity? Select all that apply. 1 Frequent usage of words such as "what" 2 Postural changes while listening to the speaker 3 Bending towards the other person while talking 4 Mismatch in the questions asked and the responses given 5 Startled expression when there is any unexpected sound in the environment

Frequent usage of words such as "what" Postural changes while listening to the speaker 3 Bending towards the other person while talking Mismatch in the questions asked and the responses given

A nurse is caring for a client who experienced a crushing chest injury. A chest tube is inserted. Which observation indicates a desired response to this treatment? 1 Increased breath sounds 2 Increased respiratory rate 3 Crepitus detected on palpation of the chest 4 Constant bubbling in the drainage collection chamber

Increased breath sounds

Propylthiouracil is prescribed for a client diagnosed with hyperthyroidism. The client asks the nurse, "Why do I have to take this medication if I am going to get radiation therapy?" What explanation does the nurse provide? 1 It binds previously formed thyroid hormones. 2 It decreases production of thyroid hormones. 3 Vascularity of the thyroid gland is decreased. 4 The need for thyroid iodine supplements is reduced.

It decreases production of thyroid hormones.

The primary healthcare provider diagnoses placenta previa in a primiparous client. What does this indicate to the nurse regarding the condition of the placenta? 1 Infarcted 2 Low-lying 3 Immaturely developed 4 Separating prematurely

Low-lying Implantation should occur in the upper third of the uterus; a low-lying placenta is termed placenta previa. Infarctions may appear on a placenta because of some interference with the blood supply; this is not related to its location within the uterus. Placenta previa indicates where the placenta is implanted and has no relationship to placental aging. Abruptio placentae, not placenta previa, is the premature separation of a normally implanted placenta

A client is brought by ambulance to the emergency department. The client's signs and symptoms are indicative of opioid overdose. What does the nurse expect the primary healthcare provider to prescribe? 1 Naloxone 2 Methadone 3 Epinephrine 4 Amphetamine

Naloxone

A client who has missed two menstrual periods tells a nurse at the prenatal clinic that the home pregnancy test was positive. Her last menstrual period began on June 18. According to Nägele's rule, what is the estimated date of birth (EDB)? 1 March 8 2 March 11 3 March 1 4 March 25

March 25

The nurse is caring for a client who has a lesion in the right upper lobe. A diagnosis of tuberculosis (TB) has been made. What are the clinical manifestations of tuberculosis? 1 Frothy sputum and fever 2 Dry cough and pulmonary congestion 3 Night sweats and blood-tinged sputum 4 Productive cough and engorged neck veins

Night sweats and blood-tinged sputum

A client tells the nurse, "A man is speaking to me from the corner of the room. Can you hear him?" How should the nurse respond? 1 "What's he saying to you? Does it make any sense?" 2 "Yes, I hear him, but I can't understand what he's saying." 3 "I don't hear him. There's no one in the corner of the room." 4 "No, I don't hear him, but is it making you uncomfortable to hear him?"

No, I don't hear him, but is it making you uncomfortable to hear him?"

The nurse is caring for a client who had a colostomy 2 days ago. Which nursing intervention is the priority? 1 Keeping an accurate record of oral fluid intake 2 Emphasizing the importance of regulating the diet to form stool 3 Teaching care of the incision and how to perform colostomy irrigations 4 Observing for drainage and the condition of the abdominal stoma

Observing for drainage and the condition of the abdominal stoma

An obese smoker complains of feeling sleepy during the daytime, waking up tired in the morning, and snoring heavily while sleeping. The client is found to have enlarged tonsils. Which condition may the client have? 1 Laryngeal trauma 2 Vocal cord paralysis 3 Obstructive sleep apnea 4 Subcutaneous emphysema

Obstructive sleep apnea

A nurse working on a mental health unit is caring for several clients who are at risk for suicide. Which client is at the greatest risk for successful suicide? 1 -Young adult who is acutely psychotic 2 -Adolescent who was recently sexually abused 3- Older single man just found to have pancreatic cancer 4 -middle-aged woman experiencing dysfunctional grieving

Older single man just found to have pancreatic cancer

The nurse is assessing the rate of involution of a client's uterus on the second postpartum day. Where does the nurse expect the fundus to be located? -1 At the level of the umbilicus -2 One fingerbreadth above the umbilicus -3 Above and to the right of the umbilicus - 4 One or two fingerbreadths below the umbilicus

One or two fingerbreadths below the umbilicus

A client is admitted to the hospital with a recurrence of chronic arterial insufficiency of the legs. Which clinical manifestations does the nurse expect to identify when performing an admission history and physical? 1 Edema of the feet and ankles 2 Reddened and painful areas on the calves 3 Pain when exercising and thickening of the toenails 4 Ulcers around the ankles and reports of a dull ache in the legs

Pain when exercising and thickening of the toenails

Methylphenidate is prescribed to treat a 7-year-old child's attention deficit-hyperactivity disorder (ADHD). The nurse understands that methylphenidate is used in the treatment of this disorder in children because it has what effect? 1 Diuretic effect 2 Synergistic effect 3 Paradoxical effect 4 Hypotensive effect

Paradoxical effect Methylphenidate, a stimulant, has an opposite effect on hyperactive children; the reason for this action is as yet totally unexplained. Although methylphenidate has a hypotensive effect, this is not why it is given to hyperactive children. Methylphenidate does not induce diuretic or synergistic effects.

A school-aged child who has just arrived from Africa has been exposed to diphtheria, and a nurse in the pediatric clinic is to administer the antitoxin. Which type of immunity does the antitoxin confer? 1 Active natural 2 Passive natural 3 Active artificial 4 Passive artificial

Passive artificial

A client is at high risk for developing ascites because of cirrhosis of the liver. How should the nurse assess for the presence of ascites? 1 Observe the client for signs of respiratory distress. 2 Percuss the client's abdomen and listen for dull sounds. 3 Palpate the lower extremities over the tibia and observe for edema. 4 Listen for decreased or absent bowel sounds while auscultating the abdomen.

Percuss the client's abdomen and listen for dull sounds

When assessing a client, the nurse auscultates a murmur at the second left intercostal space (ICS) along the sternal border. This reflects sound from which valve? 1 Aortic 2 Mitral 3 Pulmonic 4 Tricuspid

Pulmonic

Which screening report will help the nurse determine skeletal growth in a child?

Radiographs of the hand and wris

The nurse is caring for a 1-hour-old newborn. Which assessment characteristics represent a preterm gestational age? 1 Skin: thin, veins visible; breasts: flat areolae, no buds; plantar creases: absent; lanugo: abundant 2 Skin: parchment/wrinkled; breasts: flat areolae, no buds; plantar creases: cover entire sole; lanugo: absent 3 Skin: thin, veins visible; breasts: flat areolae, no buds; plantar creases: covering the entire sole; lanugo: abundant 4 Skin: cracking/few veins; breasts: raised areolae (3- to 4-mm buds); plantar creases: covering the anterior two thirds of the sole; lanugo: thinning

Skin: thin, veins visible; breasts: flat areolae, no buds; plantar creases: absent; lanugo: abundant

At 1 am a 28-month-old toddler is admitted to the pediatric unit with suspected meningitis. At 3 am, after the child is settled in, the mother tells the nurse, "I have to leave now, but whenever I try to go my child gets upset and then I start to cry." What is the best action by the nurse? 1 Walking the mother to the elevator 2 Encouraging the mother to spend the night 3 Staying with the child while the mother leaves 4 Telling the mother to wait until the child falls asleep

Staying with the child while the mother leaves

A nurse is assessing a newborn with caput succedaneum. How does the nurse explain the cause of this fetal condition to the new mother? 1-Overlap of fetal bones as they pass through the maternal birth canal 2- Swelling of the soft tissue of the scalp as a result of pressure during labor 3- Hemorrhage of ruptured blood vessels that does not cross the suture lines 4 -Accumulation of fluid resulting from partial blockage of cerebrospinal fluid drainage ✓

Swelling of the soft tissue of the scalp as a result of pressure during labor

A client has laparoscopic surgery to remove a calculus from the common bile duct. What postoperative client response indicates to the nurse that bile flow into the duodenum is reestablished? 1 Stools become brown 2 Liver tenderness is relieved 3 Colic is absent after ingestion of fats 4 Serum bilirubin level returns to the expected range

Stools become brown The return of brown color to the stool indicates that bile is entering the duodenum and being converted to urobilinogen by bacteria. Liver tenderness is unrelated to bile flow. The absence of biliary colic is related to the removal of the calculus, not the flow of bile. The serum bilirubin level is not affected.

The healthcare provider prescribes a progressive exercise program that includes walking for a client with a history of diminished arterial perfusion to the lower extremities. The nurse explains to the client what to do if leg cramps occur while walking. Which instruction did the nurse give the client? 1 Chew one aspirin twice a day. 2 Stop to rest until the pain resolves. 3 Walk more slowly while pain is present. 4 Take one nitroglycerin tablet sublingually

Stop to rest until the pain resolves.

When a client has a myocardial infarction, one of the major manifestations is a decrease in the conductive energy provided to the heart. When assessing this client, the nurse is aware that the existing action potential is in direct relationship to what? 1 Heart rate 2 Refractory period 3 Pulmonary pressure 4 Strength of contraction

Strength of contraction

An adolescent who has had type 1 diabetes for 5 years stops adhering to the therapeutic regimen. In light of the client's developmental level, what does the nurse conclude that the behavior is a reflection of? 1 Need for attention 2 Struggle for identity 3 Denial of the diabetes 4 Regression related to the illness

Struggle for identity

What should the nurse include in the plan of care for a client who just had a total laryngectomy? 1 Instructing the client to whisper 2 Removing the outer tracheostomy tube as needed 3 Placing the client in the orthopneic position 4 Suctioning the tracheostomy tube whenever necessary ✓

Suctioning the tracheostomy tube whenever necessary ✓

On reviewing the data of a client with thyroid disorder, the primary healthcare provider prescribed atenolol. Which assessment findings would indicate the need for atenolol therapy? Select all that apply. 1 Tachycardia 2 Atrial fibrillation 3 Distant heart sounds 4 Systolic hypertension 5 Decreased cardiac output

Tachycardia Atrial fibrillation Systolic hypertension

Which nursing actions reflect Leininger's caring theory in practice? 1 The nurse supports and accepts the client's feelings. 2 The nurse promotes beauty, comfort, dignity, and peace. 3 The nurse learns culturally specific behaviors to meet the client's needs. 4 The nurse believes that every caring process has subparts that should be considered while making an effective strategy.

The nurse learns culturally specific behaviors to meet the client's needs.

A client on a psychiatric unit who has been hearing voices is receiving a neuroleptic medication for the first time. The client takes the cup of water and the pill and stares at them. What is the most therapeutic statement the nurse can make? 1 "You have to take your medicine." 2 "This is the medication that your healthcare provider ordered." 3 "This will help you not to hear the voices. It will only work if you take it." 4 "There must be a reason that you don't want to take your medicine.

This will help you not to hear the voices. It will only work if you take it."

The nurse examines a client who has had a cesarean birth. It has been 3 days since the birth, and the client is about to be discharged. Where does the nurse expect the fundus to be located? 1 One fingerbreadth below the umbilicus 2 Two fingerbreadths below the umbilicus 3 Three fingerbreadths below the umbilicus 4 Four fingerbreadths below the umbilicus

Three fingerbreadths below the umbilicus

A client is to receive conscious sedation during a cardiac catheterization. Which route of administration should the nurse explain will be used to deliver the conscious sedation? 1 Via a face mask 2 Into the epidural space 3 Through an intravenous catheter 4 Around the nerves innervating the chest wall

Through an intravenous catheter

Which information may be obtained by palpation? Select all that apply. 1 Turgor 2 Bruises 3 Texture 4 Lesions 5 Moisture content 6 Tissue integrity

Turgor Texture Lesions Moisture content

A client with liver dysfunction reports bleeding gums. In addition, the nurse observes small facial hemorrhagic lesions. What should the nurse conclude that the client needs? A client with liver dysfunction reports bleeding gums. In addition, the nurse observes small facial hemorrhagic lesions. What should the nurse conclude that the client needs? 1-Vitamin C 2- Folic acid 3- Vitamin A 4 -Vitamin K

Vitamin K

A nurse notices a firm, edematous, irregularly shaped skin lesion on a client who reports an insect bite. Which skin lesion is this? 1 Wheal 2 Plaque 3 Vesicle 4 Pustule

Wheal A wheal is a firm, edematous, irregularly shaped skin lesion, formed as an inflammatory response to an allergen or insect bite. A plaque is a circumscribed, elevated, superficial lesion, like psoriasis. A vesicle is a circumscribed, superficial collection of serous fluid. A pustule is an elevated, superficial lesion filled with purulent fluid.

A registered nurse is evaluating the statements of a nursing student providing instructions to the partner of a client with Ebola. Which instruction given by the nursing student needs correction? 1 "You should avoid direct contact with your partner's saliva." 2 "You should be careful because the Ebola virus spreads through the air." 3 "You should avoid having sex with your partner for 3 months even after the recovery of your partner." 4 "You should immediately report symptoms of fever, headache, and vomiting.

You should be careful because the Ebola virus spreads through the air.

A client in a detoxification unit has an alcohol withdrawal seizure. Diazepam 7.5 mg intramuscularly stat is prescribed. Diazepam is available as 5 mg/mL. How many milliliters will the nurse administer? Record your answer using one decimal place. ___ mL

1.5

A nurse keeps track of the immunization schedules for a childbearing family. Which type of nursing intervention is executed in this situation? 1 Acute care 2 Home care 3 Health promotion 4 Restorative and continuing care

Health promotion

The nurse teaching a health awareness class identifies which situation as being the highest risk factor for the development of a deep vein thrombosis (DVT)? 1 Pregnancy 2 Inactivity 3 Aerobic exercise 4 Tight clothing

Inactivity

A client is admitted to the hospital for surgery for rectosigmoid colon cancer, and the nurse is obtaining a health history as part of the admission process. What clinical findings associated with rectosigmoid colon cancer does the nurse expect the client to report? Select all that apply. 1 Feeling tired 2 Rectal bleeding 3 Inability to digest fat 4 Change in the shape of stools 5 Feeling of abdominal bloating

Feeling tired Rectal bleeding Change in the shape of stools Feeling of abdominal bloating

Potassium supplements are prescribed for a client receiving diuretic therapy. What client statement indicates that the teaching about potassium supplements is understood? 1 "I will report any abdominal distress." 2 "I should use salt substitutes with my food." 3 "The drug must be taken on an empty stomach." 4 "The dosage is correct if my urine output increases."

"I will report any abdominal distress."

A client newly diagnosed with scleroderma states, "Where did I get this from?" How should the nurse reply? 1 "The exact cause is unknown, but it is thought to be a result of autoimmunity." 2 "The exact cause is unknown, but it is thought to be a result of ocular motility." 3 "The exact cause is unknown, but it is thought to be a result of increased amino acid metabolism." 4 "The exact cause is unknown, but it is thought to be a result of defective sebaceous gland formation."

"The exact cause is unknown, but it is thought to be a result of autoimmunity.

A healthcare provider prescribes oral loperamide (Maalox) and intravenous ranitidine for a client with burns and crushing injuries caused by an accident. The client asks how these medications work. What is the nurse's best response? 1 "They decrease irritability of the bowel." 2 "They limit acidity in the gastrointestinal tract." 3 "They are very effective in clients with multiple trauma." 4 "They work in the way that antiemetics do.

"They limit acidity in the gastrointestinal tract.

A nurse teaches a group of teenage girls about puberty. What information should the nurse refrain from providing? 1 "Your voice will change." 2 "Your shoulders will increase in width." 3 "You will grow until you are 16 to 17 years of age." 4 "Your body will gain height and weight disproportionately

"Your body will gain height and weight disproportionately.

A diabetic client survives a fire. The client has a head injury, tachycardia, and pale and ashen-colored skin. Which primary interventions should the nurse perform? Select all that apply. 1 Administering cool intravenous fluids 2 Placing the client in a cool environment 3 Managing and maintaining airway, breathing, and circulation 4 Monitoring heart rhythm, oxygen saturation, and urine output 5 Monitoring temperature, vital signs, and level of consciousness

-Administering cool intravenous fluids -Placing the client in a cool environment Managing and maintaining airway, breathing, and circulation

A pediatric client reports difficulty breathing and swallowing and has a sore throat, headache, and fever. The nurse observes a grayish yellow membranous patch near the tonsils. What disease does the nurse infer from these findings? 1 Mumps 2 Pertussis 3 Diphtheria 4 Pneumonia

-Diphtheria Diphtheria is caused by Corynebacterium diphtheriae. Symptoms include a sore throat, fever, and headache along with gray or dirty-yellow membrane patches. Mumps is a viral disease that primarily affects the parotid glands. It is accompanied by local pain, tenderness, fever, and swelling. Pertussis, also known as whooping cough, is caused by Bordetella pertussis. Symptoms include rhinorrhea, mild fever, and a persistent cough. Pneumonia is a lung disease; symptoms include a cough and chest pain.

What complications are associated with excessive weight gain during pregnancy in adolescents? Select all that apply. 1 Fetal anemia 2 Preterm labor 3 Cesarean delivery 4 Maternal mortality 5 Postpartum obesity

-Preterm labor -Cesarean delivery -Postpartum obesity

After surgery for repair of a myelomeningocele, the nurse places the infant in a side-lying position with the head slightly elevated. What is the main reason the nurse places the infant in this position after this particular surgery? 1 To prevent aspiration 2 To promote respiration 3 To reduce intracranial pressure 4 To maintain cleanliness of the suture site

-To reduce intracranial pressure The side-lying position with the head slightly elevated promotes venous return by gravity, which helps reduce intracranial pressure, a problem after myelomeningocele repair. Although preventing aspiration, promoting respiration, and maintaining cleanliness of the suture line are all important, the reason for this position that is unique with this type of surgery is that it minimizes intracranial pressure.

A client who was forced into early retirement is found to have severe depression. The client says, "I feel useless, and I've got nothing to do." What is the best initial response by the nurse? 1 "Tell me more about feeling useless." 2 "Volunteering can help you fill your time." 3 "Your illness is adding to your current feelings." 4 "Let's talk about what you'd like to be doing right now.

1 "Tell me more about feeling useless.

What statements about culturally congruent care by the student nurse are correct? Select all that apply. 1 "It is the main goal of transcultural nursing." 2 "It is provided through cultural competence." 3 "It is provided in accordance with set criteria." 4 "It is bound to the professional health care system." 5 "It depends on the patterns and needs of an individual."

1 -"It is the main goal of transcultural nursing." 2- "It is provided through cultural 5-"It depends on the patterns and needs of an individual

A nurse is caring for a variety of clients. In which client is it most essential for the nurse to implement measures to prevent pulmonary embolism? 1 A 59-year-old who had a knee replacement 2 A 60-year-old who has bacterial pneumonia 3 A 68-year-old who had emergency dental surgery 4 A 76-year-old who has a history of thrombocytopenia

1 A 59-year-old who had a knee replacement

A nurse is teaching a parent about the immunization schedule for babies. Between which months of age should the measles vaccine be given? 1 -2 and 5 2- 6 and 8 3- 9 and 11 4-12 and 15

12 and 15

The diet prescribed for a client allows for 190 grams of carbohydrates, 90 grams of fat, and 100 grams of protein. The nurse calculates that this diet contains approximately how many calories? 1 920 2 1970 3 2470 4 2970

1970

Which subdimension would form a part for the caring process "doing for" according to the Swanson's theory of caring? Select all that apply. 1 Focusing 2 Protecting 3 Comforting 4 Seeking cues 5 Generating alternatives

2 Protecting 3 Comforting

A client cannot understand how syphilis was contracted because there has been no sexual activity for several days. Which length of time associated with the incubation of syphilis should the nurse include in the teaching plan? 1-1 week 2 -4 months 3- 2 to 6 weeks 4-48 to 72 hours

2 to 6 week

What is the maximum length of time a nurse should allow an intravenous bag of solution to infuse? 1. 6 hours 2. 12 hours 3. 18 hours 4. 24 hours

24 hours

Which information is important to obtain during the nursing assessment of a female who is taking oral contraceptives? Select all that apply. 1- Family stability and socioeconomic status 2- Maternal history for estimated gestation 3 -History of vascular or thromboembolic disorder 4 -Drug interactions leading to a decreased effect of oral contraceptives 5 -Prescription of a medication that may have its therapeutic effects decreased if taken with oral contraceptives

3 -History of vascular or thromboembolic disorder- 4- Drug interactions leading to a decreased effect of oral contraceptives 5- Prescription of a medication that may have its therapeutic effects decreased if taken with oral contraceptives

A hospice nurse is caring for a dying client and the client's family members during the developing awareness stage of grief. What is the most important thing about the family that the nurse should assess before providing care? 1 Cohesiveness 2 Educational level 3 Cultural background 4 Socioeconomic status

3 Cultural background

A client is admitted to the hospital with multiple signs and symptoms associated with a cardiac problem. What clinical finding alerts the nurse that the primary healthcare provider probably will insert a pacemaker? 1 Angina 2 Chest pain 3 Heart bloc 4 Tachycardia

3 Heart block

A nurse obtains the history of a client with early colon cancer. Which clinical finding does the nurse consider consistent with a diagnosis of cancer of the descending, rather than the ascending, colon? 1 Pain 2 Fatigue 3 Anemia 4 Obstruction

4 Obstruction

While assessing a client during an ophthalmic pupillary physical examination, the primary healthcare provider observes a noticeable difference in the size of pupils in the client. Which term should the nurse use to describe this condition? 1 Mydriasis 2 Hyperopia 3 Anisocoria 4 Arcus senilis

Anisocoria

A 62-year-old client reports to the nurse, "My eyes don't feel right and I have a gritty and sandy sensation in my eyes." What condition might this client have? 1 Retinal detachment 2 Infection of the cornea 3 Changes in tear composition 4 Hemorrhage in the vitreous humor

Changes in tear composition

A client describes his delusions in minute detail to the nurse. How should the nurse respond? 1 Changing the topic to reality-based events 2 Continuing to discuss the delusion with the client 3 Getting the client involved in a social project with peers 4 Disputing the perceptions with the use of logical thinking

Changing the topic to reality-based events

A nurse understands that after the administration of alprazolam it is important to assess the client for side effects. What will the nurse do initially? 1 Measure urine output. 2 Check the blood pressure. 3 Look for abdominal distention. 4 Check the size of the pupils frequently

Check the blood pressure.

A client is diagnosed with acute kidney failure secondary to dehydration. An intravenous (IV) infusion of 50% glucose with regular insulin is prescribed. What does the nurse recognize as the primary purpose of the IV insulin for this client? 1 Correct hyperkalemia 2 Increase urinary output 3 Prevent respiratory acidosis 4 Increase serum calcium levels

Correct hyperkalemia

A 2-year-old child is admitted to the pediatric unit with a diagnosis of thalassemia major (Cooley anemia). The parents are told that there is no cure, but the anemia can be treated with frequent blood transfusions. The father tells the nurse he is glad that there is a treatment that "fixes" his child's problem. Before responding, the nurse should recall that blood transfusions do what? 1 Correct the anemia, but may cause other problems 2 Reverse the anemia, but also present a risk of hepatitis 3 Are a supportive treatment; fewer will be needed as the child grows older 4 Are a replacement for defective red blood cells; they are like giving insulin to a person with diabetes

Correct the anemia, but may cause other problems

A young client who has just lost her first job comes to the mental health clinic very upset and says, "I just start crying without any reason and without any warning." How should the nurse respond initially? 1 "Do you know what makes you cry?" 2 "Most of us need to cry from time to time." 3 "Crying unexpectedly can be very upsetting." 4 "Are you having any other problems at this time?

Crying unexpectedly can be very upsetting."

A primary healthcare provider diagnoses late-stage (tertiary) syphilis in a client. Which statement made by the client supports this diagnosis? 1 "I noticed a wart on my penis." 2 "I have sores all over my mouth." 3 "I've been having a sore throat lately." 4 "I'm having trouble keeping my balance."

I'm having trouble keeping my balance.

Which of the following drugs is commonly used as an adjunct during conscious sedation for minor surgeries? 1 Diazepam 2 Midazolam 3 Lorazepam 4 Clonazepam

Midazolam

Which drug acts as an abortifacient in female clients? 1 Mifepristone 2 Metyrapone 3 Cyproheptadine 4 Aminoglutethimide

Mifepristone

A client was administered oxytocin to induce labor, but the client was not responsive to the treatment. Which oral uterine stimulant is the next choice in this condition? 1 Misoprostol 2 Mifepristone 3 Dinoprostone 4 Methylergonovine

Misoprostol

A registered nurse (RN) must assess the body temperature of a client with a history of epilepsy. Which site for measuring temperature is contraindicated in this client? 1 Skin 2 Axilla 3 Oral cavity 4 Temporal artery

Oral cavity

A client is prescribed uterine stimulants to augment labor. Which condition should be assessed in the client before initiating therapy? 1 Liver disease 2 Pituitary tumors 3 Pelvic inflammatory disease 4 Presence of an intrauterine device

Pelvic inflammatory disease

A client with leukemia who is receiving a chemotherapeutic regimen that includes vincristine reports numbness and loss of feeling in the legs below the knees. The client asks the nurse about what is causing these problems. What fact forms the basis for the nurse's response? 1 Enlarged lymph nodes in the groin related to the cancer may cause these symptoms. 2 Most chemotherapeutic regimens do not affect the nervous or peripheral vascular system. 3 Vascular occlusion may be the cause, and immediate medical evaluation is indicated. 4 Peripheral neuropathies can result from chemotherapy and usually are reversible if promptly treated

Peripheral neuropathies can result from chemotherapy and usually are reversible if promptly treated. ✓

A client visits a primary healthcare provider with a report of burning and a sharp pain in the sole of the foot that intensifies in the morning. Which abnormal condition does the nurse anticipate in the client? 1 Torticollis 2 Pes planus 3 Tenosynovitis 4 Plantar fasciitis

Plantar fasciitis

The client diagnosed with a fistula between the bowel and urinary bladder reports passing air and bubbles during urination. What does the nurse anticipate the client's condition to be? 1 Nocturia 2 Oliguria 3 Pneumaturia 4Stress incontinence

Pneumaturia The occurrence of gas along with urination is called pneumaturia and could result from the formation of a fistula between the bowel and urinary bladder.

The nurse finds that a child has developed the qualities of independence, self-control, and self-governance. Which type of development does the nurse anticipate in the toddler? 1 Social development 2 Spiritual development 3 Biologic development 4 Psychosocial development

Psychosocial development

A nurse is caring for a client after abdominal surgery and encourages the client to turn from side to side and to engage in deep-breathing exercises. What complication is the nurse trying to prevent? 1 Metabolic acidosis 2 Metabolic alkalosis 3 Respiratory acidosis 4 Respiratory alkalosis

Respiratory acidosis

A client's cardiac monitor shows a PQRST wave for each beat and indicates a rate of 120 beats per minute. The rhythm is regular. The nurse concludes that the client is experiencing what? 1 Atrial fibrillation 2 Sinus tachycardia 3 Ventricular fibrillation 4 First-degree atrioventricular block

Sinus tachycardia The presence of a P wave before each QRS complex indicates a sinus rhythm; a heart rate greater than 100 regular beats per minute is referred to as tachycardia. Atrial fibrillation has no well-defined P waves, there are 350 or more beats per minute, there are random ventricular beats, and the rhythm is irregular. Ventricular fibrillation is irregular and shows no PQRST configurations. A firs

A client is admitted to the hospital with the diagnosis of severe anxiety. What should the nurse's plan of care for a client with an anxiety disorder include? 1 Promoting the suppression of anger by the client 2 Supporting the verbalization of feelings by the client 3 Encouraging the client to limit anxiety-related behaviors 4 Restricting the involvement of the client's family during the acute phase

Supporting the verbalization of feelings by the client Freedom to ventilate feelings serves as a safety valve to reduce anxiety. The suppression of anger may increase the client's anxiety

An older adult, accompanied by family members, is admitted to a long-term care facility with symptoms of dementia. What initial statement by the nurse during the admission procedure would be most helpful to this client? 1 "You're a little disoriented now, but don't worry. You'll be all right in a few days." 2 "Don't be afraid. I'm your nurse, and everyone here in the hospital is here to help you." 3 "I'm the nurse on duty today. You're in the hospital. Your family can stay with you for a while." 4 "Let me introduce you to the staff here first. In a little while I'll get you acquainted with our unit routine

"Don't be afraid. I'm your nurse, and everyone here in the hospital is here to help you.

A 56-year-old man is admitted to the inpatient unit after family members report that he seems to be experiencing auditory hallucinations. The man has a history of schizophrenia and has had several previous admissions. Which statement indicates to the nurse that the client is experiencing auditory hallucinations? 1 "Get these horrible snakes out of my room!" 2 "I am not the devil! Stop calling me those names!" 3 "The food on this plate has poison in it, so take it away—I won't eat it." 4 "I did see an alien spaceship last night outside in my yard, and I've felt worse ever since.

"I am not the devil! Stop calling me those names!"

A preterm newborn is admitted to the neonatal intensive care unit (NICU). Which concern is most commonly expressed by NICU parents? 1 Fear of handling the infant 2 Delayed ability to bond with the infant 3 Prolonged hospital stay needed by the infant 4 Inability to provide breast milk for the infant

1 Fear of handling the infant

During assessment of a newborn in the nursery, the nurse notices a large, dark pigmentation over the buttocks of one of the infants. What is the most important intervention? -1 Checking the infant's platelet count -2 Calling social services about potential abuse -3 Checking the medical record regarding this finding at birth -4 Notifying the provider that the infant sustained trauma at birth

Checking the medical record regarding this finding at birth

The nurse is applying skin ointment to acne lesions for a client who has recently reached puberty. What does the nurse anticipate as the cause for the extensive acne? 1 Thelarche as the first indication of puberty 2 Influence of gonadal and adrenal androgens 3 Increased subcutaneous fat before a skeletal growth spurt 4 Highly active sebaceous glands in "flush areas" of the body

Highly active sebaceous glands in "flush areas" of the body

A child is diagnosed with classic hemophilia. A nurse teaches the child's parents how to administer the plasma component factor VIII through a venous port. It is to be given three times a week. When should the parents administer this therapy? 1 Whenever a bleed is suspected 2 In the morning on scheduled days 3 At bedtime while the child is lying quietly in bed 4 On a regular schedule at the parents' convenience

In the morning on scheduled days

The nurse is caring for a client who is on a low-carbohydrate diet. With this diet, there is decreased glucose available for energy and fat is metabolized for energy, resulting in an increased production of which substance in the urine? 1 Protein 2 Glucose 3 Ketones 4 Uric acid

Ketones

A nurse is caring for a client with the diagnosis of bulimia nervosa. What does the nurse understand to be the function of food for individuals with bulimia? 1 Gain attention 2 Control others 3 Avoid growing up 4 Meet emotional needs ✓

Meet emotional needs

When monitoring fluids and electrolytes, the nurse recalls that the major cation-regulating intracellular osmolarity is what? 1 Sodium 2 Potassium 3 Calcium 4 Calcitonin

Potassium

Several hours after administering insulin, the nurse is assessing a client for an adverse response to the insulin. Which client responses are indicative of a hypoglycemic reaction? Select all that apply. 1 Tremors 2 Anorexia 3 Confusion 4 Glycosuria 5 Diaphoresis

Tremors Confusion Diaphoresis

Amitriptyline is an antidepressant medication used to treat anxiety disorders. The nurse recognizes that is it a member of which drug class? 1 Tricyclics 2 Monoamine oxidase inhibitors (MAOIs) 3 Selective serotonin reuptake inhibitors (SSRIs) 4 Serotonin-norepinephrine reuptake inhibitors (SNRIs)

Tricyclics

A pregnant client with iron-deficiency anemia is prescribed iron supplements daily. To help the client increase iron absorption, the nurse should suggest that the client eat foods high in which substance? 1 Vitamin C 2 Fat content 3 Water content 4 Vitamin B complex

Vitamin C

Which should the nurse share with the parents of a toddler-age client regarding a predominant change that occurs to the gastrointestinal system during this stage of development? 1 Food is rejected due to texture 2 Voluntary control of elimination 3 Capacity of the stomach decreases 4 Acidity of gastric contents increases

Voluntary control of elimination

The nurse caring for a 3-year-old child with meningitis should be alert for which signs and symptoms of increased intracranial pressure? Select all that apply. 1 Vomiting 2 Headache 3 Irritability 4 Tachypnea 5 Hypotension

Vomiting Headache Irritability

Which criteria should a nurse assess before preparing to discharge a pediatric client after the administration of conscious or moderate sedation? Select all that apply. 1 Check if gag reflexes are intact 2 Check if pain and ambulation are at a base level 3 Check if a reversal drug is administered just before discharge 4 Check if the oxygen saturation is below 95% on room air half an hour after the administration of the last dose 5 Check if an adult is accompanying the child and can remain with him or her for at least two half-lives of the anesthetic drug ✓

1 Check if gag reflexes are intact 2 Check if pain and ambulation are at a base level 5 Check if an adult is accompanying the child and can remain with him or her for at least two half-lives of the anesthetic drug

An infant is born with a bilateral cleft palate. Plans are made to begin reconstruction immediately. What nursing intervention should be included to promote parent-infant attachment? 1 Demonstrating positive acceptance of the infant 2 Placing the infant in a nursery away from view of the general public 3 Explaining to the parents that the infant will look normal after the surgery 4 Encouraging the parents to limit contact with the infant until after the surgery

1 Demonstrating positive acceptance of the infant

An adolescent is found to have type 1 diabetes. The nurse plans to teach the adolescent that dietary control and exercise can help regulate the disorder. What additional information should the nurse include in the teaching plan? Select all that apply . 1 Insulin therapy 2 Prophylactic antibiotics 3 Blood glucose monitoring 4 Oral hypoglycemic agents 5 Adherence to the treatment regimen

1 Insulin therapy 3 Blood glucose monitoring 5 Adherence to the treatment regimen

What are the goals of the Childhood Immunization Initiative Program? Select all that apply. 1 To reduce the costs of a vaccine 2 To involve more federal agencies 3 To improve vaccine delivery systems 4 To evaluate the impact of national immunization policies and practices 5 To prevent the occurrence of indigenous infections such as diphtheria, rubella, and measle

1 To reduce the costs of a vaccine 2 To involve more federal agencies 3 To improve vaccine delivery systems 5 To prevent the occurrence of indigenous infections such as diphtheria, rubella, and measles

How do adolescents establish family identity during psychosocial development? Select all that apply. 1 By acting independently to make his or her own decisions 2 By evaluating his or her own health with a feeling of well-being 3 By fostering his or her own development within a balanced family structure 4 By building close peer relationships to achieve acceptance in the society 5 By achieving marked physical changes

1-By acting independently to make his or her own decisions 3-By fostering his or her own development within a balanced family structure

What is the effect of parathyroid hormone on bones? Select all that apply. 1 Increased bone breakdown 2 Increased serum calcium levels 3 Increased sodium and phosphorus excretion 4 Increased absorption of calcium and phosphorus 5 Increased net release of calcium and phosphorus

1-Increased bone breakdown 2 -Increased serum calcium levels 5- Increased net release of calcium and phosphorus

Which gestational period is appropriate for the administration of corticosteroids during preterm labor? 1- Less than 20 weeks 2- 20 to 24 weeks 3 -24 to 34 weeks 4-More than 34 weeks

24 to 34 weeks

Which nursing intervention helps to prevent medication errors in children? Select all that apply 1- Encouraging the use of brand nam 2-Promoting the use of abbreviations and acronyms 3- Minimizing the use of verbal and telephone orders 4 -Carefully reading all labels for accuracy and checking expiration dates 5-Recording the client's weight before carrying out the medication orderes

3 Minimizing the use of verbal and telephone orders 4 Carefully reading all labels for accuracy and checking expiration dates 5 Recording the client's weight before carrying out the medication order

Which type of hypersensitivity reaction is present in a client with a body temperature of 102 °F, severe joint pain, rashes on the extremities, and enlarged lymph nodes from serum sickness? 1 Delayed reaction 2 Cytotoxic reaction 3 Immediate reaction 4 Immune complex-mediated reaction

4 Immune complex-mediated reaction

Which statement is true regarding varicocele? 1 It is commonly seen in prepubertal children. 2 It results in partial or complete venous occlusion. 3 It results in the red, warm, and edematous scrotum. 4 It causes elongation of the veins of the spermatic cord.

4 It causes elongation of the veins of the spermatic cord

Which nursing actions are accurate when measuring a head circumference for an infant? Select all that apply 1 Using a cloth tape for accuracy 2 Obtaining one measurement per visit 3 Documenting the information in the progress notes 4 Using paper tape marked with tenths of a centimeter 5 Placing the tape slightly above the eyebrows and pinna of the ears

4-Using paper tape marked with tenths of a centimeter 5 -Placing the tape slightly above the eyebrows and pinna of the ears

Endotracheal intubation and positive-pressure ventilation are instituted because of a client's deteriorating respiratory status. Which is the priority nursing intervention? 1 Facilitate verbal communication 2 Prepare the client for emergency surgery 3 Maintain sterility of the ventilation system 4 Assess the client's response to the mechanical ventilation

Assess the client's response to the mechanical ventilation

Which over-the-counter (OTC) drug is used as the first line of therapy for acne vulgaris in adolescents? 1- Tretinoin 2 -Isotretinoin 3 -Azithromycin 4-Benzoyl peroxide

Benzoyl peroxide

The nurse is assessing a client with severe cirrhosis and discovers fetor hepaticus. What did the nurse assess? 1 Urine 2 Stool 3 Hands 4 Breath

Breath

While assessing the skin of a light-skinned client, the nurse concludes that the client has ecchymosis. Which skin color variation would confirm this diagnosis? 1 Gray color 2 Dark red color 3 Deep brown color 4 White color

Dark red color

Which symptoms indicate to the nurse that the client has an inadequate fluid volume? Select all that apply. 1 Decreased urine 2 Hypotension 3 Dyspnea 4 Dry mucous membranes 5 Pulmonary edema 6 Poor skin turgor

Decreased urine Hypotension Dry mucous membranes Poor skin turgor Lowered urinary output, hypotension, dry mucous membranes, and poor skin turgor are all symptomatic of dehydration. Dyspnea and pulmonary edema may be caused by fluid overload.

A school-aged child is admitted to the pediatric unit with hypertensive acute glomerulonephritis. In addition to hydralazine, what medication does the nurse anticipate will be prescribed initially? 1 Digoxin 2 Furosemide 3 Alprazolam 4 Phenytoin

Furosemide

Which drug will most likely cause the client's eyelids to itch and eyes to burn as side effects? 1 Ketorolac 2 Ofloxacin 3 Diclofenac 4 Vidarabine

Vidarabine

A nurse is preparing a client for discharge from the emergency department. Which client statement provides evidence that the client understands the instructions for the prescribed high-dose ampicillin? 1 "I should take this medication with meals." 2 "I can stop taking this medication when I feel better." 3 "I will miss eating my yogurt while taking this medication." 4 "I must increase my intake of fluids while taking this medication."

"I must increase my intake of fluids while taking this medication.

A couple seeking genetic counseling are heterozygous carriers of Tay-Sachs disease. They ask the nurse what the chances are that each of their children will inherit the disease. The nurse responds that the probability is what? 1- 0% 2- 25% 3 -50% 4 -100%

25%

A client presents to the emergency department with weakness, dizziness, and difficulty breathing. The nurse performs an electrocardiogram (ECG) and notices this arrhythmia. Which arrhythmia is the client exhibiting? 1 Atrial fibrillation (AF) 2 Ventricular tachycardia (VT) 3 Junctional tachycardia 4 Supraventricular tachycardia (SVT)

Atrial fibrillation (AF)

Which breathing technique should the nurse instruct the client to use as the head of the fetus is crowning? 1 Shallow 2 Blowing 3 Slow chest 4 Modified paced

Blowing

According to Kübler-Ross, during which stage of grieving are individuals with serious health problems most likely to seek other medical opinions? 1 Anger 2 Denial 3 Bargaining 4 Depression

Denial

While caring for a pregnant client with a body mass index of 32 during labor, the nurse observes that the second stage of labor lasts for about 11 minutes. The nurse also finds that the expected birth weight of the fetus is around 4200 g. Which complication does the nurse anticipate in the neonate after birth? 1 Erb palsy 2 Klumpke palsy 3 Strawberry hemangioma 4 Erythema toxicum neonatorum

Erb palsy

A nurse is teaching a client with pruritus about personal care interventions. Which statement made by the client indicates the nurse needs to intervene? 1 "I will trim my fingernails regularly." 2 "I will wear mittens or splints at night." 3 "I will apply moisturizing lotion after bath." 4 "I will not file the edges of fingernails."

I will not file the edges of fingernails

Which is a clinical manifestation of the Landouzy-Déjérine type of muscular dystrophy (MD)? 1 Loss of hearing 2 Cardiomyopathy 3 Respiratory failure 4 Mental impairment

Loss of hearing

Which drug is responsible for neonatal hypoglycemia? 1 Warfarin 2 Simvastatin 3 Tolbutamide 4 Methimazole

-Tolbutamide Tolbutamide is an oral hypoglycemic agent used in the treatment of type 2 diabetes mellitus. It is known to have teratogenic effects like neonatal hypoglycemia

A client is admitted for treatment of partial- and full-thickness burns of the entire right lower extremity and the anterior portion of the right upper extremity. The nurse performs an immediate appraisal, using the rule of nines. Which percentage of body surface area burned will the nurse record? 1 18% 2 22.5% 3 27% 36.5%

22.5

An adult reports anxiety, palpitations, and a feeling of impending doom. After a thorough physical examination, the healthcare provider diagnoses a panic attack. Oral lorazepam 1.5 mg stat is prescribed. The lorazepam is available in 0.5-mg tablets. How many tablets will the nurse administer? Express your answer as a whole number. _____ tablet(s)

3

A nurse is taking the health history of a client who is to have surgery in one week. The nurse identifies that the client is taking ibuprofen for discomfort associated with osteoarthritis and notifies the healthcare provider. Which drug does the nurse expect will most likely be prescribed instead of the ibuprofen? 1 Naproxen 2 Aspirin 3 Ketorolac 4 Acetaminophen

Acetaminophen

A hospitalized psychiatric client with the diagnosis of histrionic personality disorder demands a sleeping pill before going to bed. After being refused the sleeping pill, the client throws a book at the nurse. What does the nurse recognize this behavior to be? 1 Exploitive 2 Acting out 3 Manipulative 4 Reaction formation

Acting out

Pharmacokinetic factors determine the concentration of a drug at its site of action. These are different in neonates and infants than they are in small children or adults. Arrange the factors in the order in which they occur. 1. Excretion 2. Absorption 3. Distribution 4. Metabolism ✓

Ans- The first step after administering the drug through any route is absorption. For instance, orally administered drugs are generally absorbed in the gastrointestinal tract. After absorption, drugs may be distributed through protein binding, or some may enter the central nervous system through the blood-brain barrier. Most drugs are then metabolized in the liver; this capacity is low in newborns. Drugs do not remain in the body indefinitely; rather they are excreted by the kidneys

A client feeling increasingly tired seeks medical care. Type 1 diabetes is diagnosed. What causes increased fatigue with type 1 diabetes? 1 Increased metabolism at the cellular level 2 Increased glucose absorption from the intestine 3 Decreased production of insulin by the pancreas 4 Decreased glucose secretion into the renal tubules

Decreased production of insulin by the pancreas Insulin facilitates transport of glucose across the cell membrane to meet metabolic needs and prevent fatigue. With diabetes there is decreased cellular metabolism because of the decrease in glucose entering the cells. Glucose is not absorbed from the intestinal tract by the cells; fatigue is caused by decreased, not increased, cellular levels of glucose. Filtration and excretion of glucose by the kidneys do not regulate energy levels; if insulin production is adequate, glucose does not spill into the urine

A client with myasthenia gravis is to receive immunosuppressive therapy. What assures the nurse that this therapy will be effective? 1 -Inhibits the breakdown of acetylcholine at the neuromuscular junction 2 -Stimulates the production of acetylcholine at the neuromuscular junction 3- Decreases the production of autoantibodies that attack acetylcholine receptors 4- Promotes the removal of autoantibodies that impair the transmission of impulses

Decreases the production of autoantibodies that attack acetylcholine receptors

A nurse who is observing a sleeping newborn at 2 hours of age identifies periods of irregular breathing and occasional twitching movements of the arms and legs. The neonate's heart rate is 150 beats/min; the respiratory rate is 50 breaths/min; and the glucose strip reading is 60 mg/dL (3.3 mmol/L). What does the nurse conclude that these findings indicate? 1 Hypoglycemia 2 Seizure activity 3 Expected adaptations 4 Respiratory distress syndrome

Expected adaptations

Before an amniocentesis, both parents express anxiety about the fetus's safety during the test. Which nursing intervention is most appropriate in promoting the parents' ability to cope? 1 Initiating a parent-primary healthcare provider conference 2 Reassuring them that the procedure is safe 3 Explaining the procedure, step by step 4 Arranging for the father to be present during the test

Explaining the procedure, step by step

The registered nurse is teaching the nursing student about the realms of family life. Which component does the registered nurse include while teaching about integrity processes? 1 Family rituals 2 Family relationships 3 Family life stressors and daily hassles 4 Family care takings and responsibilities

Family rituals

A nurse is assessing a client for possible malabsorption syndrome. Which stool assessment finding will support this diagnosis? 1 Melena 2 Frank blood 3 Fat globules 4 Currant jelly consistency

Fat globules

The school nurse presents a program on Reye syndrome to the Parent-Teacher Association. After the program the nurse talks with a group of parents. Which statement by a mother indicates the need for additional education on Reye syndrome? 1 "Aspirin should be avoided in children with viral diseases. 2 "It's OK for me to give my kids Tylenol if they run a fever." 3 "I should watch for my child's skin to turn yellow during the recovery from varicella." 4 "I need to seek medical help right away if my child starts vomiting profusely during the recovery period of a viral illness."

I should watch for my child's skin to turn yellow during the recovery from varicella

A client's membranes ruptured 20 hours before admission. The client was in labor for 24 hours before giving birth. For which postpartum complication is this client at risk? 1 Infection 2 Hemorrhage 3 Uterine atony 4 Amniotic fluid embolism

Infection

A client who retired a year ago tells the nurse in the community health center, "I don't have any hobbies or interests, and since I retired I feel useless and unneeded." According to Erikson's developmental theory, with which developmental conflict is the client faced? 1 Initiative versus guilt 2 Integrity versus despair 3 Intimacy versus isolation 4 Identity versus role confusion

Integrity versus despair

During the course of treatment a toddler is to receive an intramuscular injection. What is the priority nursing intervention that should be included in the plan of care to comfort the child? 1-Distracting the toddler's attention with a toy car 2 -Telling the parents exactly what will be done to the toddler 3- Giving the toddler the choice of having the injection now or later 4- Involving the parents in comforting the toddler after the injection

Involving the parents in comforting the toddler after the injection ✓

An adolescent who works out 6 hours a day reports not eating well, weight loss, and an absence of menses for the past few months. Which nursing intervention is most appropriate? 1 Ask the adolescent to stop exercising for a few days. 2 Talk to the client to find out any reasons for stress. 3 Perform a -human chorionic gonadotropin pregnancy test. β 4 Modify the adolescent's diet to incorporate more nutrients

Modify the adolescent's diet to incorporate more nutrients

A client with a history of type 1 diabetes is diagnosed with heart failure. Digoxin is prescribed. What is an important nursing action associated with this drug? 1- Administer the digoxin 1 hour after the client's morning insulin 2- Monitor the client for atrial fibrillation and first-degree heart block 3- Administer the medication with 8 ounces (240 mL) of orange juice 4- Withhold the medication if the apical pulse rate is greater than 60 beats/min

Monitor the client for atrial fibrillation and first-degree heart block

A client is receiving an antipsychotic medication. When assessing the client for signs and symptoms of pseudoparkinsonism, the nurse will be alert for which complication? 1 Drooling 2 Blurred vision 3 Muscle tremors 4 Photosensitivity

Muscle tremors

During a physical assessment, the nurse notes cutaneous fibromas and Lisch nodules (yellow elevations) on a client's irises. What genetic condition might this client have? 1 Phenylketonuria 2 Neurofibromatosis 3 Huntington disease 4 Myotonic dystrophy

Neurofibromatosis Cutaneous fibromas and Lisch nodules (yellow elevations on the iris) are signs of neurofibromatosis. Growth failure, frequent vomiting, irritability, hyperactivity, and erratic behavior are signs of phenylketonuria. Huntington disease is a progressive neurodegenerative disease. Muscle weakness, wasting, myotonia, and cardiac conduction abnormalities are signs of myotonic dystrophy

During a newborn assessment the nurse identifies the absence of the red reflex in the eyes. What should the nurse's next action be? 1 Rinse the eyes with sterile saline 2 Notify the primary healthcare provider 3 Expect edema to subside within a few days 4 Conclude that this is a result of the prescribed eye prophylaxis

Notify the primary healthcare provider

An older adult client who is accustomed to taking enemas periodically to avoid constipation is admitted to a long-term care facility. In addition to medications, the healthcare provider prescribes bed rest and a regular diet. Which action should be implemented initially to help prevent the client from developing constipation? 1 Arrange to have enemas prescribed for the client 2 Obtain a prescription for a daily laxative for the client 3 Place a commode by the bedside to facilitate defecation 4 Offer a large glass of prune juice with warm water each morning

Offer a large glass of prune juice with warm water each morning

An older adult who was in a motor vehicle collision is brought to the emergency department via ambulance. The client exhibits a decreased level of consciousness, and the nurse identifies serosanguineous drainage from the client's left ear. Which action should the nurse take? 1 Irrigate the ear with normal saline. 2 Place a sterile pad over the external ear. 3 Gently insert a cotton-tipped swab in the ear canal. 4 Pack a cotton ball in the external meatus of the ear

Place a sterile pad over the external ear.

What safety measure should the nurse instruct parents to follow when their child is a toddler? 1 Place window guards on all windows. 2 Have the toddler sleep on the back or side. 3 Start swimming training for the toddler under supervision. 4 Teach the child how to cross streets and walk in parking lots

Place window guards on all windows.

A client is admitted for repair of bilateral inguinal hernias. Before surgery the nurse assesses the client for indicators that strangulation of the intestine may have occurred. What is an early indicator of strangulation? 1 Increased flatus 2 Projectile vomiting 3 Sharp abdominal pain 4 Decreased bowel sound

Sharp abdominal pain

A client who sustained burn injuries due to a fire and explosion has a carbon monoxide level of 14%. Which pathophysiologic risk is increased in the client? 1 Stupor 2 Vertigo 3 Convulsions 4 Slight breathlessness

Slight breathlessness

A nurse is caring for a client who was diagnosed with a myocardial infarction. While caring for the client 2 days after the event, the nurse identifies that the client's temperature is elevated. The nurse concludes that this increase in temperature is most likely the result of what? 1 Tissue necrosis 2 Venous thrombosis 3 Pulmonary infarction 4 Respiratory infection

Tissue necrosis

A client has thin, dark-red vertical lines about 1 to 3 mm long in the nails. Which diseases are associated with this physiologic alteration in the client? Select all that apply. 1 Psoriasis 2 Trichinosis 3 Cardiac failure 4 Diabetes mellitus 5 Bacterial endocarditis

Trichinosis Bacterial endocarditis Thin, dark-red vertical lines about 1 to 3 mm long in the nails are associated with trichinosis (parasitic disease) and bacterial endocarditis (infection of the innermost layer of the heart and heart valves). Psoriasis, diabetes mellitus, and cardiac failure are associated with yellow-brown discoloration of the nails.

A newborn is being treated with phototherapy for hyperbilirubinemia. What is the nurse's role when providing phototherapy? 1 Turning the infant every 2 hours 2 Measuring the bilirubin level every 2 hours 3 Maintaining the infant on daily 24-hour phototherapy 4 Applying a sterile gauze pad to the infant's umbilical stump

Turning the infant every 2 hours The infant's position is changed every 2 hours to expose all skin surfaces to the phototherapy for maximum effect. Measuring the bilirubin level every 2 hours is not necessary. The infant may be removed from the lights for feeding and the eye patches removed to assess t

During the first session of a therapy group, one of the clients asks, "What's supposed to happen in this group?" What is the most appropriate response by the nurse facilitator? 1 "First I'd like for you to tell me what you want to happen." 2 "This is your group, and your participation will largely determine what happens." 3 "The purpose of this group is to examine the way each of you interacts with the others." 4 "You and the

"First I'd like for you to tell me what you want to happen."

A nurse is evaluating scenarios that are based on the responses of several clients. Which statement of a client confirms that he or she has reached the Intimacy versus Isolation stage according to Erikson's theory of psychosocial development? 1 "I donate a large sum of money to the local school every year." 2 "I want to enjoy my motherhood and that's why I am leaving the job." 3 "In the winter of my life, I feel that I do not have anyone to take care of me." 4 "I did so much for my partner but I was dumped for someone more attractive."

"I did so much for my partner but I was dumped for someone more attractive

One minute after birth a nurse assesses a newborn and auscultates a heart rate of 90 beats/min. The newborn has a strong, loud cry, moves all extremities well, and has acrocyanosis but is otherwise pink. What is this neonate's Apgar score? 1 9 2 8 3 7 4 6

9 no segura

A client whose membranes have ruptured is admitted to the birthing unit. Her cervix is dilated 3 cm and 50% effaced. The amniotic fluid is clear, and the fetal heart rate is stable. Which outcome does the nurse anticipate? -1 A prolonged second stage of labor -2 A difficult birth resulting from delayed effacement -3 Birth of the fetus within a day - 4 The stimulation of labor with an oxytocin infusion

Birth of the fetus within a day

A client is admitted to the mental health unit with the diagnosis of major depressive disorder. Which statement alerts the nurse to the possibility of a suicide attempt? 1 -"I don't feel too good today." 2- "I feel much better; today is a lovely day." 3 -"I feel a little better, but it probably won't last." 4-I'm really tired today, so I'll take things a little slower

"I feel much better; today is a lovely day."

A nurse educates a group of teenagers about the physical changes they will experience. What statement made by the teenager shows inadequate learning? 1 "My shoulders will change in width." 2 "My reproductive system will develop." 3 "My body fat distribution will be altered." 4 "My bones will grow at a decreased rate."

"My bones will grow at a decreased rate.

What does a community-based nurse do as a change agent? Select all that apply. 1 The nurse empowers clients and their families to creatively solve problems. 2 The nurse works with clients to solve problems and helps clients identify an alternative care facility. 3 The nurse helps clients gain the skills and knowledge needed to provide self-care. 4 The nurse empowers clients to become instrumental in creating change within a health care agency. 5 The nurse does not make decisions but rather helps clients reach decisions that are best for them.

-The nurse empowers clients and their families to creatively solve problems. -The nurse works with clients to solve problems and helps clients identify an alternative care facility. -The nurse empowers clients to become instrumental in creating change within a health care agency.

A client with a traumatic brain injury is demonstrating signs of increasing intracranial pressure, which may exert pressure on the medulla. What should the nurse assess to determine involvement of the medulla? Select all that apply. 1 Taste 2 Breathing 3 Heart rate 4 Fluid balance 5 Voluntary movement

Breathing Heart rate

A nurse is caring for a client who is admitted to the hospital with severe dyspnea and a diagnosis of cancer of the lung. What does the nurse conclude is the probable cause of the severe dyspnea? 1 Abdominal distention or pressure 2 Bronchial obstruction or pleural effusion 3 Fluid retention as a result of renal failure 4 Anxiety associated with pain on inspiration

Bronchial obstruction or pleural effusion

The nurse is performing a skin assessment of a client. Which findings in the client may indicate a risk of skin cancer? Select all that apply. 1 Lesion 2 Lumps 3 Rashes 4 Bruising 5 Dryness

1 Lesion 2 Lumps 3 Rashes

How should a nurse control the negative behavior of a toddler? 1 By allowing for more openness 2 By using invasive methods 3 By limiting his or her opportunities to say no 4 By giving the toddler choice

By limiting his or her opportunities to say no

A nurse is preparing to administer insulin to a client with diabetes. In which order should the nurse perform the actions associated with insulin administration? 1. Wipe the top of the insulin vial with an alcohol swab. 2. Wash hands with soap and water 3. Rotate the vial of insulin between the palms of the hands. 4. Withdraw the correct amount of insulin from the inverted vial. 5. Instill air into the vial of insulin equal to the desired dose.

23154

Place the pathophysiologic process of tuberculosis infection in its correct order. 1. Necrotic areas calcify or liquefy. 2. Caseation necrosis occurs in the center of the lesion. 3. Areas of caseation undergo resorption, degeneration, and fibrosis. 4. Granulomatous inflammation is created by tuberculosis bacillus in lungs. 5. Granulomatous inflammation becomes surrounded by collagen, fibroblasts, and lymphocytes.

45231 Tuberculosis is a highly communicable disease caused by Mycobacterium tuberculosis. The process of infection in tuberculosis starts by formation of granulomatous inflammation by tuberculosis bacillus in lungs. This granulomatous inflammation then becomes surrounded by collagen, fibroblasts, and lymphocytes. The necrotic tissue then turns into a granular mass, called a caseation necrosis, which occurs in the center of the lesion. Then the areas of caseation undergo resorption, degeneration, and fibrosis. Finally, the necrotic areas undergo calcification or liquefaction

A couple interested in delaying the start of a family discuss the various methods of family planning. Together they decide to use the basal body temperature method. The nurse explains that the fertile period surrounding ovulation lasts from when to when? 1 -12 hours before to 24 hours after ovulation 2 -72 hours before to 24 hours after ovulation 3- 72 to 80 hours before to 72 hours after ovulation 4 -24 to 48 hours before to 48 hours after ovulation

72 hours before to 24 hours after ovulation

A client with adrenal insufficiency reports feeling weak and dizzy, especially in the morning. What should the nurse determine is the most probable cause of these symptoms? 1 A lack of potassium 2 Postural hypertension 3 A hypoglycemic reaction 4 Increased extracellular fluid volume

A hypoglycemic reaction

The laboratory international normalized ratio (INR) results of a client receiving warfarin have been variable. The nurse interviews the client to determine factors contributing to the problem. Which is most important for the nurse to identify? 1 Use of analgesics 2 Serum glucose level 3 Serum potassium levels 4 Adherence to the prescribed drug regimen

Adherence to the prescribed drug regimen

At 12 weeks' gestation a client who is Rh negative expels the total products of conception. What is the nursing action after it has been determined that she has not been previously sensitized to Rh-positive red blood cells? 1 Administering Rho(D) immune globulin within 72 hours 2 Making certain that Rho(D) immune globulin is administered at the first clinic visit 3 Withholding the Rho(D) immune globulin because the gestation lasted only 12 weeks 4 Withholding the Rho(D) immune globulin because it is not indicated after fetal death

Administering Rho(D) immune globulin within 72 hours

A cast is applied to the involved extremity of an infant with talipes equinovarus (clubfoot). How often does the nurse tell the parents to bring their baby back to the clinic for a cast change? 1 Each week 2 Once a month 3 When the cast edges fray 4 If the cast becomes soiled

Each week

Which order of actions should a nurse follow when performing a chest examination of a client with a pulmonary disorder? 1 . Monitor the respiratory rate. 2. Observe for any abnormalities of the sternum. 3. Observe for any evidence of respiratory distress. 4.4. Observe the shape and symmetry of the chest.

Ans- The first step is to observe the client`s appearance and note any evidence of respiratory distress. Next, the shape and symmetry of the chest along with the chest movements should be determined. After this step, the nurse should check for any abnormalities of the sternum such as pectus carinatum and pectus excavatum. Finally, the nurse should observe for any respiratory rates and abnormal breathing patterns such as Kussmaul breathing or Cheyne-Stokes respiration.

While assessing the development of a child, the nurse finds that the child is imitating the actions of her elders. Which stage, according to Erickson's developmental theory, does the nurse expect the child is in? 1 Initiative versus guilt 2 Industry versus inferiority 3 Identity versus role confusion 4 Autonomy versus shame and doubt

Autonomy versus shame and doubt

A nurse educates a parent about Erikson's theory of child development. At what age would the nurse advise the parent to begin instilling a sense of trust in his or her child? 1 -Birth to 1 year 2 -1 to 3 years 3 -3 to 6 years - 4 -6 to 11 years

Birth to 1 year

A client is admitted to the hospital with atrial fibrillation. A diagnosis of mitral valve stenosis is suspected. The nurse concludes that it is most significant if the client presents with what history? 1 Cystitis as an adult 2 Pleurisy as an adult 3 Childhood strep throat 4 Childhood German measles

Childhood strep throat

The nurse is caring for a client who underwent intestinal surgery 3 days ago and notices brownish pus with a fecal odor draining from the incision. What should the nurse infer from this finding? 1 Colonization with Proteus 2 Colonization with Pseudomonas 3 Colonization with Staphylococcus 4 Colonization with aerobic coliform and Bacteroides

Colonization with aerobic coliform and Bacteroides

A client with mild preeclampsia is admitted to the high-risk prenatal unit because of a progressive increase in her blood pressure. The nurse reviews the primary healthcare provider's prescriptions. Which prescriptions does the nurse expect to receive for this client? Select all that apply. 1 Daily weight 2 Side-lying bed rest 3 2 g/day sodium diet 4 Deep tendon reflexes 5 Glucose tolerance test

Daily weight Side-lying bed rest Deep tendon reflexes

A client developed acute herpes zoster and was treated with antiviral medication within 72 hours of the appearance of the rash. The client is reporting persistent pain 1 week later. What does the nurse identify as the cause of the posttherapeutic neuralgia? 1 Damage to the nerves 2 Untreated major depression 3 Scarring in the area of the rash 4 Continued presence of the skin rash

Damage to the nerves

The nurse is caring for a 70-year-old client who presents with dilute urine even when fluid intake is low. What could be the possible cause of the client's condition? 1 Decreased glucose tolerance 2 Decreased general metabolism 3 Decreased ovarian production of estrogen 4 Decreased antidiuretic hormone production

Decreased antidiuretic hormone production

While assessing the client's skin, a nurse notices a skin condition, the pathophysiology of which involves increased visibility of oxyhemoglobin caused by an increased blood flow due to capillary dilation. Which condition is associated with this client? 1 Pallor 2 Vitiligo 3 Cyanosis 4 Erythema

Erythema

What is an important aspect of nursing care for a client exhibiting psychotic patterns of thinking and behavior? 1 Helping keep the client oriented to reality 2 Involving the client in activities throughout the day 3 Helping the client understand that it is harmful to withdraw from situations 4 Encouraging the client to discuss why interacting with other people is being avoided

Helping keep the client oriented to reality

A client who is in the first trimester is being discharged after a week of hospitalization for hyperemesis gravidarum. She is to be maintained at home with rehydration infusion therapy. What is the priority nursing activity for the home health nurse? 1 Determining fetal well-being 2 Monitoring for signs of infection 3 Monitoring the client for signs of electrolyte imbalances 4 Teaching about changes in nutritional needs during pregnancy

Monitoring the client for signs of electrolyte imbalances

A nurse is assessing a client admitted to the hospital with a tentative diagnosis of a pituitary tumor. What signs of Cushing syndrome does the nurse identify? 1 Retention of sodium and water 2 Hypotension and a rapid, thready pulse 3 Increased fatty deposition in the extremities 4 Hypoglycemic episodes in the early morning

Retention of sodium and water Hypoglycemic episodes in the early morning ✓ Ans- 1 Increased levels of steroids and aldosterone cause sodium and water retention in clients with Cushing syndrome. Hypertension, not hypotension, is expected because of sodium and water retention. The extremities will be thin; subcutaneous fat deposits occur in the upper trunk, especially the back between the scapulae. Hyperglycemia, not hypoglycemia, occurs because of increased secretion of glucocorticoids. Hyperglycemia is sustained and not restricted to the morning hours.

An older client experiences a cerebral vascular accident (CVA) and has right-sided hemiplegia and expressive aphasia. The client's children ask the nurse which functions will be impaired. Which abilities does the nurse explain will be affected? 1 Stating wishes verbally 2 Recognizing familiar objects 3 Comprehending written words 4 Understanding verbal communication

Stating wishes verbally

A client is admitted to the hospital with a diagnosis of lower extremity arterial disease (LEAD) or peripheral arterial disease. Which is the most beneficial lifestyle modification the nurse should teach this client? 1 Stop smoking 2 Control blood glucose 3 Start a walking program 4 Eat a low-fat, low-cholesterol diet

Stop smoking

The nurse is administering medication through an implanted port. What nursing safety priority should the nurse follow in this scenario? 1 The nurse should use barrel syringes to flush any central line. 2 The nurse should use 20 mL of sterile saline to flush the port after drawing blood. 3 The nurse should use 10 mL of sterile saline to flush the port before and after medication administration 4 The nurse should withhold the drug until patency and adequate noncoring needle placement of the port are established

The nurse should withhold the drug until patency and adequate noncoring needle placement of the port are established.

The nurse is asking a client with arthritis questions in order to collect information. Which questions asked by the nurse are closed-ended questions? Select all that apply. 1 "Are you having pain?" 2 "Tell me how your pain has been." 3 "Describe how your husband is helping you at home." 4 "Do you think the medication is helping you to get pain relief?" 5 "Give me an example of a method which helps you to get pain relief at home

"-Are you having pain? -Do you think the medication is helping you to get pain relief?"

While teaching a prenatal class about infant feeding, the nurse is asked about the relationship between breast size and ease of breastfeeding. How should the nurse respond? 1 "Breast size is not related to milk production." 2 "Motivated women tend to breastfeed successfully. 3 "You seem to have some concerns about breastfeeding." 4 "Glandular tissue in the breasts determines the amount of milk you'll produce.

"Breast size is not related to milk production.

What is most appropriate for a nurse to say when interviewing a newly admitted depressed client whose thoughts are focused on feelings of worthlessness and failure? 1 "Tell me how you feel about yourself." 2 "Tell me what has been bothering you." 3 "Why do you feel so bad about yourself?" 4 "What can we do to help you while you're here?

"Tell me how you feel about yourself."

The parents of a 2-year-old child tell the nurse that they are having difficulty disciplining their child. What is the most appropriate response by the nurse? 1 "This is a difficult age that your child is going through right now." 2 "Tell me more about your difficulty. I'm not sure what you mean." 3 "It's important to be consistent with toddlers when they need discipline." 4 "I totally understand what you mean

"Tell me more about your difficulty. I'm not sure what you mean."

A client is admitted for a biopsy of a tumor in her left breast. The client states, "I know it can't be cancer, because it doesn't hurt." What is the nurse's most therapeutic response? 1 "Let's hope that it isn't malignant." 2 "What do you know about breast cancer?" 3 "Most lumps in the breast are not malignant." 4 "Has your primary healthcare provider told you that it wasn't cancer?"

"What do you know about breast cancer?

What primary nursing actions are used to prevent pediatric medication errors? Select all that apply. 1 Carefully decipher illegibly written orders. 2 Have full knowledge of the medication, including its on- and off-label uses. 3 Assume that the prescriber is correct and administer the medications as ordered. 4 Always confirm information about the client before delivering a dose. 5 Make sure all the orders are clear and well understood during a shift change.

-Have full knowledge of the medication, including its on- and off-label uses. -Always confirm information about the client before delivering a dose. -Make sure all the orders are clear and well understood during a shift change

A client is admitted to the postanesthesia care unit after a segmental resection of the right lower lobe of the lung. A chest tube drainage system is in place. When caring for this tube, what should the nurse do? 1 Raise the drainage system to bed level and check its patency 2 Clamp the tube when moving the client from the bed to a chair 3 Mark the time and fluid level on the side of the drainage chamber 4 Secure the chest catheter to the wound dressing with a sterile safety pin

-Mark the time and fluid level on the side of the drainage chamber. The fluid level and time must be marked so that the amount of drainage in the chest tube drainage system can be evaluated. The drainage system must be kept below chest level to promote drainage of the pleural space so the lungs can expand. Clamping the tube can produce backpressure, which may cause fluid to move into the pleural space from which it came, producing a tension pneumothorax. The catheter is secured by skin sutures, not to a dressing with a safety pin.

Which criteria in a client must be assessed in order to diagnose premenstrual dysphoric disorder (PMDD)? Select all that apply. 1 Symptom-free period in the luteal phase 2 Symptoms that resolve within a few days of menses onset 3 Symptoms caused by the exacerbations of other disorder 4 Symptoms such as irritability, anxiety, or depressed mood 5 Symptoms that interfere with work or interpersonal relationships

4 -Symptoms such as irritability, anxiety, or depressed mood 5 -Symptoms that interfere with work or interpersonal relationships

The nurse is assessing a term newborn. Which sign should the nurse report to the pediatric primary healthcare provider? 1 Temperature of 97.7° F (36.5° C) 2 Pale-pink to rust-colored stain in the diaper 3 Heart rate that decreases to 115 beats/min 4 Breathing pattern with recurrent sternal retractions

4 Breathing pattern with recurrent sternal retractions

A client diagnosed with asthma has received a prescription for an inhaler. The nurse teaches the client how to determine when the inhaler is empty, instructing the client to do what? 1 Count the number of doses taken. 2 Taste the medication when sprayed into the air. 3 Shake the canister. 4 Place the canister in water to see if it floats.

Count the number of doses taken


Conjuntos de estudio relacionados

Chapter 27: Common Reproductive Conditions

View Set

Lesson 3: Work Relationships + Patient Interactions

View Set

Chapter 3 Self Regulation and Ego Depletion

View Set

Insurance Regulation quiz study set

View Set

AD Banker Life & Health Chapter 6

View Set

Networking Essentials - Chapter 1 Test

View Set

EVR FINAL REVIEW chp. 19 Air Pollution

View Set